Sei sulla pagina 1di 92

PERTANYAAN

DIRECTIONS: Each question below contains five suggested responses. Select the ONE BEST
response to each question.

1. Each of the following heart sounds may occur shortly after S2 EXCEPT:

A. Opening snap

B. Third heart sound

C. Ejection click

D. Tumor plop

E. Pericardial knock

2. A murmur is auscultated in an 18-year-old asymptomatic college student at the second left


intercostal space, close to the sternum. The murmur is crescendo-decrescendo in shape, is present
throughout systole and diastole, and peaks simultaneously with S2. It does not change with position or
rotation of the head. Which of the following best describes this murmur?

A. This is a continuous murmur, most likely a venous hum commonly heard in adolescents

B. This is a continuous murmur, resulting from mixed aortic valve disease

C. This is a continuous murmur due to a congenital shunt, likely a patent ductus arteriosus

D. Continuous murmurs of this type can only be congenital; murmurs due to acquired
arteriovenous connections are purely systolic

E. This murmur, the result of left subclavian artery stenosis, is not considered continuous, as a
continuous murmur can result only from an arteriovenous communication

3. Unequal upper extremity arterial pulsations are commonly found in each of the following
disorders EXCEPT:

A. Aortic dissection

B. Takayasu’s disease

C. Supravalvular aortic stenosis

D. Subclavian artery atherosclerosis

1
E. Subvalvular aortic stenosis

4. All of the following statements about pulsus paradoxus are true EXCEPT:

A. A reduction in the systolic arterial pressure of up to 8 mm Hg during inspiration is within normal


limits and reflects a small reduction in LV stroke volume and a transmission of negative intrathoracic
pressure to the aorta

B. Pulsus paradoxus is observed frequently in cardiac tamponade

C. Pulsus paradoxus is observed in patients with pulmonary disease associated with wide swings
in intrathoracic pressure

D. In the presence of aortic regurgitation, pulsus paradoxus is less likely to develop, despite the
presence of tamponade

E. Pulsus paradoxus may occur in hypertrophic cardiomyopathy (HCM)

5. Which of the following electrocardiographic features is typical of left anterior fascicular block?

A. Q waves in the inferior leads

B. Mean QRS axis between 0 and –30 degrees

C. QRS duration >0.12 msec

D. rS pattern in the inferior leads and qR pattern in lateral leads

E. Marked right axis deviation

6. True statements about the blood pressure response during exercise testing include all of the
following EXCEPT:

A. Normal subjects will display a progressive increase in systolic blood pressure to a peak level
between 160 and 220 mm Hg

B. Black subjects tend to have higher systolic blood pressure responses than white subjects

C. An elevation of diastolic blood pressure >20 mm Hg is a relatively sensitive indicator for


underlying myocardial or coronary disease

D. HCM may lead to a fall in systolic blood pressure during exercise

2
E. Postexercise hypotension is less suggestive of severe underlying coronary artery disease than
is exertional hypotension

7. Each of the following statements regarding splitting of the second heart sound is true EXCEPT:

A. Severe pulmonic valvular stenosis is associated with a softened P2

B. Delayed closure of the pulmonic valve with inspiration contributes to physiologic splitting of S 2

C. Fixed splitting of S2 is the auscultatory hallmark of an ostium secundum atrial septal defect

D. Paradoxical splitting of S2 is expected in patients with a right ventricular electronically paced


rhythm

E. Right bundle branch block is associated with paradoxical splitting of S2

8. A 56-year-old asymptomatic man with a history of hypertension and hypercholesterolemia is


referred for a screening exercise treadmill test. After 7 minutes on the standard Bruce protocol he is
noted to have 1 mm of flat ST segment depression in leads II, III, and aVF. He stops exercising at 9
minutes because of leg fatigue and breathlessness. The peak heart rate is 85 percent of the
maximum predicted for his age. The ST segments return to baseline by 1 minute into recovery. Which
of the following statements is correct?

A. This test is conclusive for severe stenosis of the proximal right coronary artery

B. His risk of death due to an acute myocardial infarction (MI) during the next year is greater than
50 percent

C. He should be admitted for immediate cardiac catheterization

D. The test predicts a 25 percent risk of cardiac events over the next 5 years, most likely the
development of angina

E. This is likely a false positive test

9. In which of the following clinical scenarios do ST depressions during standard exercise testing
increase the diagnostic probability of significant coronary artery disease?

A. A 56-year-old man with left bundle branch block and a family history of premature coronary
disease

B. A 45-year-old woman with diabetes and hypertension, with left ventricular hypertrophy on her
baseline ECG

3
C. A 76-year-old woman with new exertional dyspnea, a history of cigarette smoking, and a
normal baseline ECG

D. A 28-year-old woman with pleuritic left-sided chest pain following a gymnastics class

E. A 63-year-old man with exertional dyspnea on beta-blocker, digoxin, and nitrate therapy

10. Each of the following statements regarding cardiac catheterization is true EXCEPT:

A. The overall mortality of cardiac catheterization is less than 1 percent

B. The incidence of contrast-induced nephrotoxicity in patients with renal dysfunction is most


effectively decreased with intravenous mannitol administration before and after the procedure

C. Low-osmolar nonionic contrast agents have a reduced incidence of adverse hemodynamic


reactions compared with high-osmolar ionic contrast agents

D. One French unit (F) is equivalent to 0.33 mm

E. Retrograde left heart catheterization should not be performed in patients with tilting-disc
prosthetic aortic valves

11. A 75-year-old woman was brought to the catheterization laboratory in the setting of an acute MI.
She presented with chest pain, epigastric discomfort, and nausea. Physical examination was pertinent
for diaphoresis, bradycardia (heart rate = 52 beats per minute), hypotension (blood pressure = 85/50),
jugular venous distension, and slight bilateral pulmonary rales. Coronary angiography demonstrated
ostial occlusion of a dominant right coronary artery, without significant left-sided coronary artery
disease. The presenting ECG likely showed all of the following features EXCEPT:

A. ST elevation in leads II, III, and aVF

B. ST depression in leads V1–V2

C. Sinus bradycardia

D. ST elevation in lead V4R

E. PR segment depression

12. Using Doppler ultrasound methods, the following values are obtained for a patient with a
restrictive ventricular septal defect (VSD) and mitral regurgitation: systolic mitral flow velocity = 5.8
m/sec, systolic flow velocity at the site of the VSD = 5.1 m/sec. The patient’s blood pressure is
144/78. The estimated right ventricular systolic pressure is (choose the single best answer):

4
A. 35 mm Hg

B. 40 mm Hg

C. 45 mm Hg

D. 50 mm Hg

E. 55 mm Hg

13. Which of the following statements regarding left bundle branch block (LBBB) is true?

A. The majority of patients with LBBB do not have structural heart disease

B. In LBBB the second heart sound is widely split with normal respiratory variation

C. The presence of LBBB is associated with significantly reduced long-term survival

D. In LBBB the T wave vectors are oriented in the same direction as the QRS complex

E. LBBB does not impair myocardial performance

14. Each of the following statements concerning cyanosis is true EXCEPT:

A. The bluish discoloration of cyanosis results from either an increased amount of reduced
hemoglobin or the presence of abnormal hemoglobin pigments

B. Central cyanosis is characterized by decreased arterial oxygen saturation

C. Patients with marked polycythemia become cyanotic at higher levels of arterial oxygen
saturation than patients with normal hematocrit

D. Peripheral cyanosis most commonly results from impaired pulmonary function

E. Localization of peripheral cyanosis to a single extremity may suggest arterial or venous


obstruction

15. Each of the following statements regarding cardiac hemodynamics is true EXCEPT:

A. The x descent of the right atrial pressure waveform represents relaxation of the atrium and
downward pulling of the tricuspid annulus by right ventricular contraction

5
B. In the left atrium, in contrast to the right atrium, the v wave is more prominent than the a wave

C. A prominent y descent is found in constrictive pericarditis and restrictive cardiomyopathy

D. Atrial ischemia can result in blunting of the x descent

E. Tricuspid stenosis results in a prominent y descent

16. For which of the following scenarios is the diagnostic sensitivity of standard exercise testing
sufficient to forego additional imaging with either nuclear scintigraphy or echocardiography?

A. A 53-year-old woman with hypertension and left ventricular hypertrophy who has developed
exertional chest pressure

B. A 74-year-old man with two prior myocardial infarctions and left ventricular dysfunction, on
angiotensin converting enzyme (ACE) inhibitor, beta-blocker, and digoxin therapy

C. A 37-year-old asymptomatic woman with incidentally detected LBBB

D. A 44-year-old male smoker with Wolff-Parkinson-White syndrome and a positive family history
of coronary artery disease with new exertional chest discomfort

E. A 53-year-old man with hyperlipidemia, a normal baseline ECG, and sharp, fleeting chest
pains

17. All of the following statements about the ECG depicted in (Fig. 1) are correct EXCEPT:

A. The basic rhythm is atrial fibrillation

B. The abnormal beat is an example of the Ashman phenomenon

C. The Ashman phenomenon is based on the fact that the refractory period is directly related to
the length of the preceding R-R interval

D. Right bundle branch block morphology is more commonly associated with this type of
aberrancy

6
E. Because the bundle of His has the longest refractory period, it is the likely anatomical location
of the conduction delay

18. Normal (innocent) murmurs are usually which type of murmur?

A. Early systolic

B. Presystolic

C. Midsystolic

D. Holosystolic

E. Early diastolic

19. Which of the following statements about the jugular venous waveform are true? Answer true (T)
or false (F):

A. The Kussmaul sign may be present in either constrictive pericarditis or congestive heart failure

B. The c wave is a reflection of ventricular diastole and becomes visible in patients with diastolic
dysfunction

C. The x descent is more prominent than the y descent in cardiac tamponade

D. Phasic declines in venous pressure (the x and y descents) are typically more prominent to the eye
than the positive pressure waves (the a, c, and v waves)

E. Cannon a waves are diagnostic of atrial tachycardia

20. Each of the following statements regarding the measurement of cardiac output is true EXCEPT:

A. In the thermodilution method, cardiac output is directly related to the area under the
thermodilution curve

B. The thermodilution method tends to overestimate cardiac output in low-output states

C. In the presence of tricuspid regurgitation, the Fick technique is preferred over the
thermodilution method for measuring cardiac output

D. A limitation of the Fick method is the necessity of measuring oxygen consumption in a steady
state

E. Cardiac output is inversely related to systemic vascular resistance

7
21. Which of the following conditions is associated with the Doppler transmitral inflow pattern shown
in the figure (Fig. 2)?

A. Gastrointestinal hemorrhage

B. Constrictive pericarditis

C. Normal aging

D. Restrictive cardiomyopathy

E. Hyperthyroidism

22. A 71-year-old woman is hospitalized because of a non-Q wave MI. On the fourth hospital day
she is asymptomatic, ambulating around the hospital floor. Her medications include aspirin, a beta-
blocker, an ACE inhibitor, and an HMG CoA reductase inhibitor. She undergoes predischarge
exercise testing, and is able to complete 9 minutes on a modified Bruce protocol (5 METS) without
symptoms or ST segment changes. Her heart rate increases from 62 to 104 during exercise (70
percent of maximal predicted heart rate), and her blood pressure rises from 120/72 to 162/74. Which
of the following statements is true?

A. The test is invalid because of inability to reach target heart rate; the beta-blocker should have
been discontinued prior to testing

B. She is ready for discharge and can return to her usual light housekeeping

C. Since she suffered a non-Q wave infarction, her risk of subsequent events is high and she
should undergo cardiac catheterization for further risk stratification

D. The exercise test should be repeated with nuclear imaging

E. Isosorbide dinitrate should be added to her pharmacological regimen

8
23. The Doppler tracing in Fig. 3 permits accurate assessment of each of the following EXCEPT:

A. The presence of mitral stenosis

B. The presence, but not the severity, of mitral regurgitation

C. The transmitral diastolic pressure gradient

D. The cause of the valvular lesion

E. The mitral valve area

24. A patient with a long history of pulmonary emboli undergoes noninvasive assessment of cardiac
status by Doppler echocardiography. Assuming the following values:

RA pressure = 4 mm Hg

Peak Doppler flow signal across the tricuspid valve = 4 m/sec

The patient’s right ventricular systolic pressure is:

A. 64 mm Hg

B. 68 mm Hg

C. 50 mm Hg

D. 20 mm Hg

9
E. Insufficient information to determine the value

25. Which of the following statements is true regarding the response to aerobic exercise in healthy
older adults?

A. Ventricular stroke volume decreases with age such that there is an age-related fall in cardiac
output during exercise

B. Systolic and diastolic blood pressures each rise significantly during aerobic exercise

C. A decline in beta-adrenergic responsiveness contributes to a fall in the maximum heart rate in


older individuals

D. A normal adult’s cardiac output doubles during maximum aerobic exercise

E. Maximum aerobic capacity does not change significantly with age in sedentary individuals

26. Physiological states and dynamic maneuvers alter the characteristics of heart murmurs. Which
of the following statements is correct?

A. In acute mitral regurgitation, the left atrial pressure rises dramatically so that the murmur is
heard only during late systole

B. Rising from a squatting to a standing position causes the murmur of mitral valve prolapse to
commence later in systole

C. The diastolic rumble of mitral stenosis becomes more prominent during the strain phase of a
Valsalva maneuver

D. The murmur of aortic stenosis, but not mitral regurgitation, becomes louder during the beat
following a premature ventricular contraction

E. The murmur of acute aortic regurgitation can be heard throughout diastole

27. All of the following are true regarding the echocardiographic assessment of mitral regurgitation
(MR) EXCEPT:

A. Color flow Doppler echocardiography provides an accurate quantitative assessment of MR

B. Pulse Doppler can provide an indirect assessment of MR

C. Regurgitant color flow Doppler jets directed toward the atrial wall underestimate the severity of
MR

10
D. Echocardiography may be used to assess the hemodynamic consequences of MR

E. Determining the etiology of MR is one of the most important applications of echocardiography

28. Findings easily shown on both two-dimensional and M-mode echocardiography in mitral
stenosis include all of the following EXCEPT:

A. Doming of the mitral valve leaflets

B. Inadequate separation of the anterior and posterior leaflets of the valve during diastole

C. Increased thickness of the valve leaflets

D. The presence of fibrosis and calcifications as revealed by an increase in the number of echoes

E. An increase in left atrial size due to left atrial hypertension

29. Which of the following statements regarding ST segment changes during exercise testing is
true?

A. The ECG localization of ST segment depression predicts the anatomic territory of coronary
obstructive disease

B. The J point is the proper isoelectric reference point on the ECG

C. J point depression during exercise is diagnostic for significant cardiac ischemia

D. Persistence of ST depression for 60 to 80 msec after the J point is necessary to interpret the
ECG response as abnormal

E. ST segment depression must be present both during exercise and in recovery to be


interpreted as abnormal

30. The ECG depicted in Fig. 4 is consistent with

11
A. Reversal of limb leads

B. Left posterior hemiblock

C. Right ventricular hypertrophy

D. Counterclockwise rotation and left anterior hemi-block

E. None of the above

31. Which of the following statements is true regarding exercise protocols?

A. Regardless of the exercise protocol, in order to achieve a valid test the heart rate and systolic
and diastolic blood pressures must all increase substantially

B. A fall in systolic blood pressure during exercise is associated with more severe coronary artery
disease

C. The standard Bruce protocol is characterized by relatively small increases in oxygen


consumption between stages

D. Bicycle, treadmill, and arm ergometry protocols all produce approximately equal heart rate and
blood pressure responses

E. An optimal graded treadmill exercise test lasts approximately 5 minutes

12
32. Which of the following patients is LEAST likely to have a cardiac cause of his/her recent onset of
dyspnea?

A. An active 54-year-old man with a congenitally bicuspid aortic valve who has recently noticed
shortness of breath walking his usual 18 holes of golf

B. A 70-year-old woman who sustained an anterior MI 1 year ago with a left ventricular ejection
fraction of 50 percent at that time. She has not had recurrent angina but has noted dys-pnea during
her usual housework over the past 2 months

C. A 46-year-old woman with a history of asymptomatic rheumatic mitral stenosis who recently
noticed irregular palpitations and shortness of breath while climbing stairs

D. A 38-year-old woman with a small ostium secundum atrial septal defect, now 8 months
pregnant, who has noted shortness of breath during her usual weekly low-impact aerobics class

E. A 22-year-old man with trisomy 21 and a heart murmur noticed shortness of breath carrying
grocery bundles over the past 3 months

33. Which of the following ECG findings is LEAST likely in a patient experiencing an acute anterior
Q-wave MI

A. Normal ECG at presentation and 3 hours later

B. ST elevation in leads V2–V5

C. New RBBB

D. ST depression in leads III and aVF

E. Hyperacute T waves across the precordium

34. All of the following statements regarding nuclear imaging and acute MI are true EXCEPT:

A. The size of the resting myocardial perfusion defect after acute MI correlates with the patient’s
prognosis

201
B. Increased lung uptake of Tl at rest correlates with an unfavorable prognosis

C. Submaximal exercise imaging prior to discharge from the hospital is a better predictor of late
complications from acute MI than adenosine or dipyridamole myocardial perfusion imaging

D. 99mTechnetium-labeled compounds, such as sestamibi, can be used to assess the


effectiveness of thrombolytic therapy

13
E. Measuring infarct size by 99mTc-sestamibi prior to discharge from the hospital is a reliable way
to predict subsequent ventricular remodeling

35. Which of the following statements regarding ST segment elevation during exercise is true?

A. ST segment elevation commonly occurs during exercise-induced myocardial ischemia

B. ST segment elevation in a lead with a pathologic Q wave at baseline indicates severe


ischemia

C. The ECG leads manifesting ST elevation localize the anatomic regions of ischemia

D. ST elevation that develops during exercise is usually a manifestation of benign early


repolarization

E. ST segment elevation during exercise is commonly associated with the development of


complete heart block

36. All of the following are true of prognosis as determined by myocardial perfusion imaging
EXCEPT:

A. Patients with normal perfusion in the presence of angiographically documented coronary


artery disease have low rates of cardiac events

B. The prognostic predictive value of myocardial perfusion is independent of imaging technique


or isotope used

C. The sensitivity of a high-risk pattern of myocardial perfusion is approximately 70 percent

D. The combination of clinical and cardiac catheterization data provides more prognostic
information than the combination of clinical and myocardial perfusion data

E. Both the number and extent of myocardial perfusion defects and the magnitude of defect
reversibility are predictors of future cardiac event

37. All of the following statements concerning radiographic contrast agents are true EXCEPT:

A. Ionic contrast-induced nephrotoxicity occurs in about 1 to 2 percent of patients receiving ionic


contrast media

B. Nonionic, low osmolar contrast agents cause fewer acute adverse hemodynamic and
arrhythmic side effects than ionic agents

14
C. There is no advantage of low osmolar contrast over ionic contrast in the prevention of
nephrotoxicity in patients with normal renal function

D. Saline hydration is less effective than furosemide and mannitol for prevention of contrast-
induced nephrotoxicity in patients with baseline renal insufficiency

E. Diabetes mellitus is an indication for using low osmolar contrast agents

38. Each of the following statements regarding intracardiac shunts is true EXCEPT:

A. A left-to-right shunt should be suspected if the difference in oxygen saturation between the
superior vena cava and the pulmonary artery is 8 percent or more

B. Oxygen saturation in the inferior vena cava (IVC) is normally higher than that in the superior
vena cava (SVC)

C. In a suspected atrial septal defect with left-to-right flow, mixed venous O2 content should be
measured at the level of the pulmonary artery

D. A pulmonic-to-systemic blood flow ratio of less than 1 indicates a net right-to-left shunt

E. Pulmonary artery oxygen saturation exceeding 80 percent should raise the suspicion of a left-
to-right shunt

DIRECTIONS: Each question below contains suggested answers. For EACH of the alternatives, you
are to respond either TRUE (T) or FALSE (F). In a given item, ALL, SOME, OR NONE OF THE
ALTERNATIVES MAY BE CORRECT.

40. Which of the following statements regarding echocardiography in pericardial disease are true?

A. Small pericardial effusions tend to accumulate anterior to the heart, adjacent to the right ventricular
free wall

B. Approximately 15 ml of pericardial fluid is present in normal individuals

C. In cardiac tamponade, right ventricular diastolic collapse may be absent in the presence of
pulmonary hypertension

D. In the presence of a pericardial effusion, right atrial diastolic indentation is a less specific sign of
cardiac tamponade than early diastolic collapse of the right ventricle

E. Transthoracic echocardiography is superior to chest computed tomography as a means to


accurately measure pericardial thickness

41. For each of the following statements about nuclear imaging in cardiac disease, indicate true (T)
or false (F):

15
A. Similar to exercise ECG testing, stress myocardial perfusion imaging is less accurate in women

B. Patients with complete left bundle branch block should be studied with dipyridamole or adenosine,
rather than exercise, to avoid artifactual perfusion defects

C. Nuclear myocardial perfusion imaging has been found to be a useful modality for preoperative
assessment of cardiac risk for both cardiac and noncardiac surgery

D. It has been shown that outcomes are similar in diabetics compared to nondiabetics for any given
myocardial perfusion defect

E. Viability of non-contracting myocardium can be accurately evaluated by thallium-201 imaging

42. Which of the following statements concerning the echocardiographic evaluation of aortic
stenosis are true?

A. The peak instantaneous pressure gradient agrees well with the peak-to-peak gradient measured at
cardiac catheterization

B. It is not necessary to calculate an aortic valve area if the peak aortic flow velocity is between 2 and
3 m/sec, since this never represents significant aortic stenosis

C. Among echocardiographic-Doppler techniques, the most accurate transaortic valve flow velocity in
aortic stenosis is determined by pulsed wave Doppler

D. The greatest degree of error in the calculation of aortic valve area using the continuity equation
resides in inaccurate measurement of the left ventricular outflow tract diameter

E. The mean aortic valve gradient measured by Doppler echocardiography is invariably higher than
the mean gradient measured by cardiac catheterization

43. With reference to intracardiac shunts, which of the following statements are true?

A. In normal subjects, O2 content in different portions of the right atrium may vary by as much as 2
volumes percent (20 ml O2/L), reflecting that streaming of blood received from the SVC, the IVC, and
the coronary sinus occurs in the right atrium

B. Atrial septal defect, anomalous pulmonary venous drainage, and ruptured sinus of Valsalva may all
lead to significant step-up in O2 saturation values between the venae cavae and the right atrium

C. Because of the normal variability in O2 saturation, shunts with pulmonary-to-systemic flow ratios
(Qp/Qs) 1.3 at the level of the pulmonary artery or right ventricle may escape detection by oximetry
run analyses

D. When a shunt is unidirectional (e.g., left-to-right only), its magnitude is calculated as the difference
between the pulmonary and systemic blood flows as determined using the appropriate variations of
the Fick equation

E. In patients with a pure right-to-left shunt, such as in tetralogy of Fallot, the calculated right-to-left
shunt will be a negative value

44. Which of the following findings of an exercise test are associated with multivessel (or left main)
coronary artery disease?

16
A. Early onset of ST segment depression

B. Persistence of ST segment changes late into the recovery phase

C. ST segment elevation in lead aVR

D. Multifocal premature ventricular contractions

E. Failure to increase systolic blood pressure by at least 10 mm Hg

45. Which of the following statements about the Doppler flow tracing in Fig. 6 are true?

A. The probability of critical aortic stenosis in this patient is high

B. The estimated aortic valve gradient is 60 mm Hg

C. The aortic insufficiency is not severe

D. Based on the Doppler findings, one would expect to find premature closure of the mitral valve on
M-mode echocardiographic examination of this patient

E. One would expect the two-dimensional (2D) echocardiogram of this patient to reveal left ventricular
hypertrophy

46. Which of the following imaging techniques are capable of correctly identifying acutely necrotic
myocardial tissue?

A. 99mTc-pyrophosphate scintigraphy

B. Echocardiography

17
C. Monoclonal antimyosin-specific antibody scintigraphy

D. Coronary angiography

E. Positron emission tomography (PET)

47. Which of the following statements regarding abnormalities of the extremities in cardiac
conditions are true? Respond true (T) or false (F).

A. Arachnodactyly is frequently associated with Marfan’s syndrome

B. A thumb with an extra phalanx commonly occurs in Turner’s syndrome

C. Quincke’s sign is typical of aortic regurgitation

D. Janeway lesions are small, tender hemorrhagic lesions on the hands and feet in patients with
infective endocarditis

E. Differential cyanosis is typical of patent ductus arteriosus with a reversed shunt

48. Which of the following statements are true regarding the echocardiographic evaluation of
suspected infective endocarditis?

A. After successful antibiotic therapy, previously detected vegetations should not be visible by
echocardiography

B. Bacterial vegetations are most commonly located on the downstream, lower-pressure side of a
valve

C. Serial echocardiograms should be obtained during antibiotic therapy, even if clinical improvement
is evident

D. Functional and structural consequences of valvular infection are rarely observed by transthoracic
echocardiographic evaluation, and transesophageal study is mandatory

E. When endocarditis is suspected, the absence of vegetations on transthoracic echocardiogram is


reassuring and should turn the diagnostic evaluation elsewhere

49. Which of the following statements regarding the arterial pulse are true? Respond true (T) or
false (F).

A. A reduced carotid pulse with a late systolic peak is characteristic of severe aortic stenosis

B. A bisferious pulse is characterized by a systolic and then a diastolic peak and is typical of mixed
mitral valve disease

C. The carotid artery is the blood vessel used to best appreciate the contour, volume, and consistency
of the peripheral vessels

D. In coarctation of the aorta, there is a distinct difference in the examination of the brachial and
femoral pulses, with the lower extremity pulses having smaller volumes and later peaks

E. The abdominal aorta is normally palpable both above and below the umbilicus

18
50. Correct statements regarding coronary angiography include which of the following? Respond
true (T) or false (F).

A. The risk of retroperitoneal hemorrhage is decreased when the femoral artery puncture is made
proximal to the inguinal ligament

B. Protamine can be administered to reverse the anticoagulation effect of unfractionated heparin

C. Mild to moderate contrast-induced allergic reactions occur in up to 9 percent of patients

D. Patients with a history of contrast allergy should not undergo coronary angiography

E. Coronary arteriography was first performed in 1959

51. Which of the following statements are true regarding the use of cardiopulmonary exercise
testing in patients with congestive heart failure? Respond true (T) or false (F).

A. A peak oxygen consumption of 14 ml/kg/min is equivalent to a peak exercise capacity of 4 METS


and the inability to complete stage I of a standard Bruce protocol

B. Patients with ejection fractions less than 20 percent consistently have peak oxygen consumptions
less than 10 ml/kg/min, and exercise testing is of little utility in this population

C. The exercise limitation in severe heart failure is due primarily to an inability to raise the heart rate

D. Exercise training in congestive heart failure patients results in decreased functional capacity

E. Results of exercise testing are rarely used when making clinical decisions in heart failure patients,
such as timing for cardiac transplantation

52. Which of the following statements concerning the echocardiographic findings in HCM are true?

A. The presence of systolic anterior motion of the mitral valve is a sensitive and specific finding in
HCM

B. Early systolic closure of the aortic valve, while indicative of significant outflow obstruction, is not a
specific sign of HCM

C. Asymmetrical septal hypertrophy (ASH) is defined echocardiographically as a ratio of septal-to-


posterior wall thickness of 1.3:1.0

D. The absence of ASH excludes the diagnosis of HCM

E. The Doppler mitral inflow pattern is typically normal in HCM

53. Which of the following associations between altered electrolytes and ECG abnormalities are
true?

A. Hypocalcemia results in prolongation of the QT interval

B. Hyperkalemia causes QRS widening and diminished P wave amplitude

C. Hypermagnesemia is associated with polymorphic ventricular tachycardia

19
D. Hypokalemia causes peaked T waves

E. Hypercalcemia leads to sinus tachycardia

54. True statements about the effects of maneuvers on auscultation of cardiac murmurs include
which of the following? Respond true (T) or false (F).

A. In patent ductus arteriosus, the diastolic phase of the murmur is intensified by isometric hand grip

B. The murmur of hypertrophic obstructive cardiomyopathy becomes softer with standing or during a
Valsalva strain maneuver

C. The murmur of a VSD increases with isometric handgrip

D. Sudden squatting or isometric handgrip increases the diastolic murmur of aortic regurgitation

E. The middiastolic and presystolic murmurs of mitral stenosis become louder with exercise

55. Which of the following statements about diastolic murmurs are true (T) and which are false (F)?

A. Diastolic murmurs are classified according to their time of onset as early diastolic, middiastolic, or
late diastolic

B. In aortic regurgitation due to aortic root dilatation, the murmur typically radiates to the right sternal
border

C. It is impossible to differentiate the murmur of acute aortic regurgitation from that of chronic aortic
regurgitation

D. The presence of a late diastolic (presystolic) murmur indicates that the patient is in atrial fibrillation

E. The Graham Steell murmur begins in early diastole after a loud P2

56. Which of the following statements regarding coronary artery anatomy are true? Respond true
(T) or false (F).

A. The left main coronary artery is best visualized in the anteroposterior (AP) projection with slight
caudal angulation

B. A ramus intermedius branch is present in more than 25 percent of people

C. The left circumflex artery is the dominant vessel in 15 percent of people

D. The most densely vascularized area of the heart is the interventricular septum

E. Coronary artery fistulae are the most common coronary congenital abnormalities that are
hemodynamically significant

57. True statements about continuous murmurs include which of the following? Respond true (T) or
false (F).

A. Continuous murmurs and holosystolic murmurs are synonymous

20
B. By definition, a continuous murmur must continue without interruption through S1

C. Patent ductus arteriosus is associated with a continuous murmur

D. A continuous cervical venous hum is commonly found in healthy children

E. Arterial continuous murmurs occur in both constricted and nonconstricted arteries

58. Which of the following echocardiographic findings suggest that aortic regurgitation is severe?

A. Diastolic flow reversal in the descending thoracic aorta

B. Premature closure of the mitral valve

C. Pressure half-time of the aortic regurgitation Doppler spectrum of 500 msec

D. A color Doppler regurgitant jet extends to the tips of the papillary muscles

E. The left ventricular outflow tract gradient is 64 mm Hg

59. Which of the following statements regarding pharmacological agents used in myocardial
perfusion stress testing are true? Respond true (T) or false (F).

A. Patients who cannot perform exercise can be evaluated for coronary artery disease (CAD) with
vasodilating medications

B. Dipyridamole blocks the cellular uptake of adenosine, an endogenous vasodilator

C. Administration of adenosine or dipyridamole results in substantial myocardial ischemia in patients


with CAD

D. Radiopharmaceutical agents should be injected 1 to 2 minutes prior to the end of exercise

E. Dobutamine is an alternative pharmacological stress testing agent for patients with


contraindications to adenosine or dipyridamole

60. True statements about digitalis-induced arrhythmias include all of the following EXCEPT:

A. Atrial tachycardia with block is diagnostic of digitalis toxicity

B. Nonparoxysmal junctional tachycardia may be a sign of digitalis excess

C. Ventricular bigeminy with varying morphology and regular coupling may be a sign of digitalis
toxicity

D. Atrioventricular (AV) dissociation in a patient taking digitalis is a strong indication of digitalis toxicity

E. Ventricular premature contractions are not highly specific for toxicity in the patient on digitalis
therapy

61. Conditions that may mimic the pain of myocardial ischemia include which of the following?
Respond true (T) or false (F).

21
A. Esophageal spasm

B. Aortic dissection

C. Herpes zoster

D. Bronchiectasis

E. Costal chondritis

62. Which of the following statements regarding exercise testing in symptomatic patients are true
(T) and which are false (F)?

A. Patients with chronic ischemic heart disease should undergo exercise testing before angiography
unless specific contraindications exist

B. In the presence of chronic stable angina pectoris, an excellent exercise tolerance is not as precise
a predictor of prognosis as the anatomical extent of coronary artery disease

C. Symptomatic patients who are unable to exercise past Bruce stage I and who exhibit 1 mm or
more of ST-segment depression have an annual mortality of 5 percent or more based on the CASS
study

D. In the presence of documented coronary artery disease, exercise-induced ischemic ST-segment


changes predict increased risk even in patients who do not experience chest pain (i.e., with silent
ischemia)

E. In the CASS study, patients stratified by coronary anatomy and left ventricular function had similar
survival rates regardless of whether their ischemia was silent or symptomatic

63. Which of the following statements regarding cardiac arrhythmias and conduction disturbances
during exercise testing are true and which are false?

A. Ventricular arrhythmias most often appear in the recovery period

B. Patients with arrhythmias who develop QT interval prolongation during exercise testing are likely to
benefit from type IA antiarrhythmic drugs

C. The appearance of sustained supraventricular tachycardia during exercise testing is diagnostic of


underlying myocardial ischemia

D. Patients who develop exercised-induced LBBB have a threefold increase in subsequent cardiac
morbidity and mortality

E. Tachyarrhythmias are commonly precipitated during exercise testing in patients with Wolff-
Parkinson-White syndrome

64. True statements concerning ejection sounds include:

A. Ejection sounds of aortic origin are most prominent in association with a deformed aortic valve,
such as a bicuspid valve, or in congenital or rheumatic aortic stenosis (AS)

B. While aortic ejection sounds due to a dilated aortic root may be less prominent than those
associated with valvular disease, they have a similar timing early in systole

22
C. A decrease in intensity of the pulmonic ejection sound with inspiration is heard in pulmonic valve
stenosis

D. It can be shown by echophonocardiography that high-frequency ejection sounds start before the
aortic or pulmonic valve is completely open

65. Which of the following statements regarding the ECG in chronic obstructive lung disease with
secondary right ventricular hypertrophy are correct? Respond true (T) or false (F).

A. The mean QRS axis is typically >110 degrees

B. The amplitude of the QRS complex is increased in the limb leads

C. Even mild right ventricular hypertrophy produces marked ECG abnormalities

D. A deep S wave in V6 is typical

E. Delayed transition in the precordial leads may simulate anterior MI

66. Which of the following conditions are often associated with a prominent R wave in
electrocardiographic lead V1? Respond true (T) or false (F).

A. Right ventricular hypertrophy

B. Wolff-Parkinson-White syndrome

C. Duchenne muscular dystrophy

D. Left anterior fascicular block

E. Misplacement of the chest leads

67. The cardiac catheterization tracing in Fig. 7 could be associated with the following features:

A. A large systolic pressure gradient between the left ventricle and aorta

23
B. A bifid aortic pulse contour

C. Increased ventricular stiffness resulting in an elevated left ventricular end-diastolic pressure

D. A slow and delayed rise in the aortic pressure as compared with that of the left ventricle

68. Which of the following statements about the second heart sound (S2) are true (T) and which are
false (F)?

A. The pulmonic component of S2 normally precedes the aortic component

B. Fixed splitting of S2 is associated with atrial septal defects

C. Physiological splitting of S2 results mainly from the earlier timing of A2 following inspiration

D. A2 is typically louder than P2

E. Paradoxical splitting of S2 occurs in LBBB and right ventricular paced rhythms

69. Which of the following bedside maneuvers are useful to distinguish the murmur of mitral valve
prolapse from other valvular lesions? Respond true (T) or false (F).

A. Isometric handgrip

B. Standing from a supine position

C. Carotid sinus massage

D. Valsalva maneuver

E. Lean forward and exhale

70. Rotation of the heart within the chest cavity results in which of the following changes in the ECG?

A. A “horizontal” heart leads to a QRS complex in aVL that resembles that in leads V5 and V6

B. “Clockwise rotation” causes the “rS” portions of the QRS complex to be present in leads V2 to V5

C. A “vertical” heart leads to a QRS complex in lead aVF that resembles that in leads V5 and V6

D. “Counterclockwise rotation” can mimic right ventricular hypertrophy

71. Which of the following statements regarding exercise testing is true?

A. A patient with a 50 percent stenosis of the right coronary artery will likely develop ischemic ST
changes in ECG leads II, III, and aVF at a peak rate-pressure product less than 15,000 mm Hg ·
beats/min

B. J-point depression always indicates myocardial ischemia

C. The electrocardiographic QT interval normally lengthens with exercise

D. Failure of systolic blood pressure to rise during exercise is a common response in normal

24
E. Maximal predicted heart rate is approximately 220 minus the patient’s age

72. Which of the following statements concerning the cardiac catheterization laboratory evaluation of
valve orifice areas are true?

A. Valve area in cm2 is calculated as: Flow (in ml/sec) ÷ K X (mean pressure gradient in mm Hg)1/2,
where K is an empirical constant for the valve in question

B. The presence of valvular regurgitation will result in a falsely low calculated valve area because
actual flow across the valve is greater than the flow calculated from the systemic cardiac output

C. Calculation of mitral valve area often relies on substitution of a confirmed pulmonary capillary
wedge pressure for left atrial pressure

D. Valve area calculation is more strongly influenced by errors in the pressure gradient measurement
than by errors in cardiac output measurement

73. A 54-year-old man who underwent coronary artery bypass grafting 4 years ago has ST and T
wave abnormalities on his baseline ECG. During exercise myocardial perfusion imaging, he stopped
at 5 minutes (stage II) of the standard Bruce treadmill protocol because of fatigue. The accompanying
99mTc-sestamibi images (Fig. 8) demonstrate which of the following? Respond true (T) or false (F).

A. No perfusion defects

B. A partially reversible defect of the entire inferior wall

C. Severe reversible defect of the anterior wall

D. A fixed defect of the anterior wall

74. Which of the statements below concerning the ECG in MI are true (T) and which are false (F)?

25
A. In the majority of patients presenting early with an MI, the initial ECG is nondiagnostic

B. The time of onset of Q waves varies between several hours and 1 or 2 days

C. The classic pattern of ECG evolution in MI begins with T wave changes

D. Infarction of an electrocardiographically silent area of the heart is one of several explanations for a
normal ECG in patients with evolving MI

E. ST segment depression in leads V1 to V4 in the presence of an inferior infarction is usually an


indication of ischemia in the distribution of the left anterior descending artery

75. Which of the following statements regarding the effect of the potassium concentration on the ECG
are true and which are false?

A. The earliest ECG sign of hyperkalemia is a reduction in P wave amplitude

B. Deep symmetrical T wave inversions are characteristic of early hyperkalemia

C. Hyperkalemia predisposes to digitalis-induced tachyarrhythmias

D. Hypokalemia is associated with torsades de pointes

E. QRS widening is common in severe hyperkalemia

76. Computed tomography of the pericardium can delineate or differentiate which of the following
features?

A. Loculated pericardial effusions and hemopericardium from transudative fluid

B. Congenital absence of the pericardium

C. Constrictive pericarditis with thickened pericardium from restrictive cardiomyopathy

D. Mucoid-secreting pericardial cysts from solid tumors

77. Which of the following conditions can result in significant electrocardiographic Q waves in the
absence of infarction? Respond true (T) or false (F).

A. Cerebrovascular accident

B. Left ventricular dilation with posterior rotation of the heart

C. ECG lead misplacement

D. Acidosis

E. Wolff-Parkinson-White syndrome

DIRECTIONS: The group of questions below consists of lettered headings followed by a set of
numbered items. For each numbered item select the ONE lettered heading with which it is MOST

26
closely associated. Each lettered heading may be used ONCE, MORE THAN ONCE, OR NOT AT
ALL.

For each of the chest roentgenograms shown (Fig. 9. A–D), match the most appropriate
cardiac diagnosis:

See Fig. 9A,See Fig. 9B,See Fig. 9C,See Fig. 9D

27
78. Mitral stenosis

79. Aortic regurgitation

80. Atrial septal defect

81. Pericardial effusion

28
Match each of the following clinical scenarios to the most likely cause of syncope:

A. Ventricular tachycardia

B. High-degree AV block

C. Epilepsy

D. Neurocardiogenic syncope

E. Hysterical fainting

82. A 73-year-old man with a remote history of MI feels the onset of palpitations while driving, then
awakens having driven his car into a ditch, unaware of what has transpired

83. A 25-year-old woman on chronic antiseizure medication becomes warm, diaphoretic, and very
pale after donating blood, then suffers frank syncope while seated upright in a chair. After being
helped to the floor, she awakens embarrassed and alert

84. A 73-year-old woman with recent episodes of dizziness begins to feel lightheaded while seated at
church, then within seconds turns pale and slumps to the floor with a few clonic jerks. She regains
consciousness 1 minute later, completely aware of where she is and asks what has happened. When
an ambulance arrives, her heart rate is 48 beats per minute

85. A 32-year-old man with a history of prior syncope notices an odd odor, after which he falls to the
ground. He awakens 3 minutes later confused and disoriented and is found to be incontinent of urine

86. An 18-year-old army recruit falls to the ground while standing at attention for 20 minutes during his
first week of basic training. He immediately awakens, feels a bit groggy, but quickly is able to rejoin
his squad

Match the clinical scenario with the most appropriate diagnostic test:

A. Rest and redistribution thallium imaging

B. Transesophageal echocardiography

C. Exercise echocardiography

D. Standard treadmill exercise test

E. Coronary angiography

87. A 53-year-old man with acute onset of severe “ripping” chest discomfort radiating to his upper
back. His ECG demonstrates low limb lead voltage and 0.5 to 1 mm ST elevation in leads II, III, and
aVF

88. A 74-year-old woman with exertional chest discomfort and dyspnea; her ECG shows a left bundle
branch block pattern

29
89. A 46-year-old man who is hypertensive and smokes cigarettes has had sharp chest discomfort
after eating for the past 4 months; his baseline ECG is normal

90. A 64-year-old man who has a history of a large MI and known three-vessel disease. His ejection
fraction is 30 percent with severe regional wall motion abnormalities, including anterior akinesis; his
ECG shows anterior and inferior Q waves

For each arteriogram or set of arteriograms (Fig. 10 A–E), match the appropriate descriptive
phrase:

30
31
91. Right anterior oblique (RAO) projection: left anterior descending (LAD) artery, demonstrating
myocardial bridging with narrowing in systole and near-normal caliber in diastole

92. Left anterior oblique (LAO) projection: right coronary arteriogram demonstrating anomalous origin
of the left circumflex artery from the proximal right coronary artery (RCA)

93. LAO projection: collateral vessels arising from the distal RCA and supplying an occluded LAD
artery

94. RAO projection: catheter-induced coronary spasm and restoration of normal caliber with
introduction of nitroglycerin

95. LAO projection: showing early filling of a markedly dilated left circumflex artery and subsequent
coronary sinus opacification due to a congenital fistula

For each of the following diuretic agents, match the appropriate adverse effect:

A. Ototoxicity

B. Gynecomastia

C. Metabolic acidosis

D. Hypercalcemia

96. Acetazolamide

97. Metolazone

98. Ethacrynic acid

99. Spironolactone

32
For each clinical scenario, select the appropriate ECG from Fig. 11 A–D:

33
100. A 49-year-old man with chronic renal failure and progressive fatigue

101. A 37-year-old man with a recent viral syndrome and sharp anterior chest pain that worsens when
he changes position

102. A 59-year-old man with severe lightheadedness

103. A 38-year-old woman with perioral and peripheral cyanosis, digital clubbing, and a history of
cardiac surgery as a child

34
ELECTROCARDIOGRAMS

DIRECTIONS: Each of the 12-lead ECGs below is introduced by a brief descriptive phrase. For each
ECG, perform a systematic reading. Begin by noting any atrial, AV junctional, or ventricular rhythms
present and point out whether any AV conduction abnormalities or atrial-ventricular interactions exist.
Determine whether criteria are met for abnormal voltage, ventricular hypertrophy, or intraventricular
conduction disturbances. Continue by noting abnormal ST and T wave changes as well as any Q
wave MI that may be apparent. Conclude by citing any suggested clinical abnormality compatible with
each tracing.

104. An elderly man was seen in a nursing home with a complaint of occasional dizziness (Fig. 12)

105. A 71-year-old man with symptoms of dizziness and shortness of breath in the emergency
department (Fig. 13)

35
106. A 54-year-old woman with weight loss (Fig. 14)

107. An exercise tolerance test tracing from a 46-year-old man (Fig. 15)

108. A 23-year-old woman with a history of a heart murmur seen in a cardiology clinic (Fig. 16)

36
109. A 29-year-old man with a history of a heart transplant seen in clinic (Fig. 17)

110. An 18-year-old woman in the emergency room with chest pain (Fig. 18)

37
111. A 20-year-old man with a heart murmur (Fig. 19)

112. A 35-year-old woman with shortness of breath seen in the cardiology clinic (Fig. 20)

38
113. An 86-year-old woman with an extensive history of cigarette smoking presents to the emergency
department with shortness of breath (Fig. 21)

114. An 18-year-old woman with a history of syncope (Fig. 22)

39
115. An 18-year-old woman with dyspnea and cyanosis (Fig. 23)

116. An 85-year-old woman during her routine clinic appointment (Fig. 24)

40
117. A 15-year-old boy with episodes of palpitations (Fig. 25)

118. A 44-year-old asymptomatic woman having a routine evaluation (Fig. 26)

41
119. A 63-year-old man with symptoms of palpitations (Fig. 27)

120. A 58-year-old man with a history of a prior MI presents to the emergency department with
symptoms of palpitations (Fig. 28)

42
121. A 21-year-old woman with symptoms of palpitations and presyncope (Fig. 29)

122. A 72-year-old man seen in cardiology clinic with chronic dyspnea on exertion (Fig. 30)

43
123. A young man in the cardiology clinic (Fig. 31)

124. A 56-year-old man in the coronary care unit (CCU) (Fig. 32)

44
125. A 42-year-old man in the emergency room with palpitations (Fig. 33)

126. A 72-year-old man in the CCU (Fig. 34)

45
127. An asymptomatic 52-year-old man (Fig. 35)

128. A 75-year-old man with symptoms of exertional dizziness (Fig. 36)

46
129. A 43-year-old woman presents to the emergency department with symptoms of palpitations
(Fig.37)

DIRECTIONS: Each question below contains five suggested responses. Select the ONE BEST
response to each question.

130. Each of the following statements regarding arrhythmias is true EXCEPT:

A. The prevalence of premature ventricular complexes increases with age

B. In the absence of structural heart disease, detection of premature ventricular complexes has
no impact on survival

C. Type IC antiarrhythmic agents are the drugs of choice for suppression of premature ventricular
complexes following myocardial infarction (MI)

D. Most concealed accessory pathways are located between the left ventricle and the left atrium

E. A concealed accessory pathway should be suspected in narrow-complex tachycardias when


the retrograde P wave occurs after completion of the QRS complex

131. All of the following statements about the management of heart failure are true EXCEPT:

A. Mannitol is a useful diuretic for patients with decompensated heart failure

B. Carbonic anhydrase inhibitors improve the alkalemia caused by other diuretic agents

C. Angiotensin-converting enzyme (ACE) inhibitors may result in hyperkalemia

47
D. Loop diuretic agents often result in hypokalemia and metabolic alkalosis

E. The effectiveness of loop diuretic agents is reduced by nonsteroidal anti-inflammatory drugs

132. A 56-year-old man with a history of hypercholesterolemia and smoking is referred to the
cardiology clinic for preoperative risk assessment prior to an orthopedic procedure. He denies any
cardiovascular symptoms and exercises regularly. His electrocardiogram (ECG) is shown (Fig. 38). All
of the following statements are true EXCEPT:

A. This syndrome is thought to account for 40 to 60 percent of all cases of idiopathic ventricular
fibrillation

B. Genetic mutations in the sodium channel have been identified in some families with this
syndrome

C. Implantable cardiac defibrillators are appropriate therapy for preventing sudden death

D. Antiarrhythmic therapy with procainamide reliably prevents ventricular dysrhythmias in this


syndrome

E. Screening of family members is recommended

133. All of the following statements are true EXCEPT:

A. The major determinants of cardiac output are heart rate and ventricular stroke volume

B. The myocardium extracts oxygen from blood nearly maximally at rest such that the coronary
sinus oxygen saturation is <40 percent

C. During strenuous exercise, oxygen consumption can increase up to 18-fold

D. The maximal O2 consumption (· O2max) and the anaerobic threshold as measured by


cardiopulmonary exercise testing are highly reproducible when measured days or weeks apart

E. A measured · O2max of 20 ml/min/kg at exercise testing indicates severe functional


impairment

134. True statements regarding the prevention of sudden cardiac death include all of the following
EXCEPT:

A. The therapy of choice for survivors of sudden cardiac death is implantation of a cardioverter-
defibrillator

48
B. The use of class I antiarrhythmic drugs to suppress ventricular ectopy after MI is associated
with an adverse outcome

C. Among survivors of out-of-hospital cardiac arrest not associated with an MI, the risk of
recurrent cardiac arrest at 1 year is about 30 percent

D. Implantable cardioverter defibrillators are the preventive therapy of choice in patients with the
Brugada syndrome

E. Amiodarone is the most appropriate therapy for patients with hypertrophic cardiomyopathy
with a prior history of syncope

135. True statements regarding atrioventricular (AV) blocks include all of the following EXCEPT:

A. In first-degree AV block, the intensity of the first heart sound is increased

B. The conduction abnormality in Mobitz type I second-degree heart block with normal QRS
duration is almost always at the level of the AV node, proximal to the His bundle

C. In Mobitz type I second-degree heart block, the absolute increase in conduction time
decreases progressively over subsequent beats

D. In Mobitz type II second-degree heart block, the PR intervals are constant prior to the
nonconducted P wave

E. The ventricular escape rate in acquired complete heart block is usually less than 40 beats/min

136. True statements about the use of Holter monitoring in the detection of cardiac arrhythmias
include all of the following EXCEPT:

A. Wenckebach second-degree AV block may be seen in normal subjects

B. The frequency of premature ventricular contractions after myocardial infarction increases over
the first several weeks

C. Holter monitoring has proved useful in the detection of potentially serious arrhythmias in
patients with mitral valve prolapse

D. The predictive ability of Holter monitoring is comparable to that of electrophysiological testing


for the determination of antiarrhythmic drug efficacy

E. In normal subjects, the cardiac rhythm detected by ambulatory monitoring shows little variation
from one recording period to the next

49
137. All of the following statements about the auscultatory findings in mitral stenosis are correct
EXCEPT:

A. The opening snap is an early diastolic sound

B. A long A2-OS interval implies severe mitral stenosis

C. In atrial fibrillation the A2-OS interval varies with cycle length

D. The “snap” is generated by bowing of the anterior mitral leaflet

E. The presence of an opening snap implies a mobile mitral valve or anterior mitral leaflet

138. A 9-year-old girl is brought in for evaluation because of several episodes of fainting. During one
episode, which occurred while she was reading a book with her mother, she turned blue and was
resuscitated. Her past medical history is unremarkable except for congenital deafness. The family
history is remarkable for a sister who died suddenly at the age of 3 years. The most likely diagnosis
is:

A. Sudden infant death syndrome

B. Jervell and Lange-Nielsen syndrome

C. Romano-Ward syndrome

D. Lown-Ganong-Levine syndrome

E. Barlow’s syndrome

139. All of the following statements regarding syncope are true EXCEPT:

A. Ten to 20 percent of syncopal episodes are of cardiac origin

B. Syncope of cardiac origin is associated with a 30 percent 1-year mortality

C. The most common causes of syncope are vascular in origin, including reflex-mediated
syncope and orthostatic hypotension

D. Supraventricular tachycardia is the most common tachyarrhythmia that leads to syncope

E. Cessation of cerebral blood flow leads to loss of consciousness in approximately 10 seconds

50
140. Which of the following conditions is likely to precipitate symptomatic heart failure in patients with
previously compensated left ventricular (LV) dysfunction?

A. Atrial fibrillation

B. Marked sinus bradycardia

C. Dissociation between atrial and ventricular contraction

D. Ventricular tachycardia

E. All of the above

141. True statements about permanent pacemakers include all of the following EXCEPT:

A. The second position in pacemaker nomenclature refers to the chamber being sensed

B. The presence of first-degree AV block with left anterior fascicular block in an asymptomatic
patient is not an indication for pacemaker placement

C. Rate-adaptive pacemakers are designated by the letter R in the fourth position of the
pacemaker code

D. Medically refractory hypertrophic cardiomyopathy is a class I indication for the placement of a


permanent dual-chamber pacemaker

E. A pacemaker mode switching option is beneficial for patients with paroxysmal supraventricular
rhythm disturbances

142. All the following characteristics are typical of hypertensive crisis EXCEPT:

A. Diastolic blood pressure >140 mm Hg

B. Retinal hemorrhages

C. Normal mental status

D. Proteinuria and azotemia

E. Microangiopathic hemolytic anemia

51
143. Which of the following statements regarding the therapy of heart failure is correct?

A. Digoxin therapy decreases the rate of hospitalizations for patients with chronic heart failure

B. ACE inhibitors improve survival in heart failure more than the combination of hydralazine plus
isosorbide dinitrate

C. Angiotensin II receptor blockers (ARBs) should be used only as second line agents for
patients who are intolerant of ACE-inhibitor therapy

D. Spironolactone reduces mortality in patients with class III-IV heart failure symptoms

E. All of the above

144. A 63-year-old man who has been an insulin-requiring diabetic for 10 years presents to the office
for initial management of hypertension (180/100). Urine and serum chemistries at this time are normal
except for serum creatinine, 1.8 mg/dl, and blood urea nitrogen (BUN), 30 mg/dl. Because of
gastroparesis you elect to initiate therapy with a potassium-sparing diuretic. When he returns in 2
weeks his serum potassium is 6.8 mmol/L with no significant change in BUN or creatinine. The most
likely explanation is:

A. Consumption of tomatoes and bananas

B. A recent urinary tract infection

C. Primary hypoaldosteronism

D. Hyporeninemic hypoaldosteronism

E. Tuberculous adrenal hypoplasia

145. Each of the following statements about cardiac transplantation is true EXCEPT:

A. Younger patients have better survival rates following cardiac transplantation

B. Use of the immunosuppressant agent cyclosporine has led to improvement in results of


cardiac transplantation

C. The majority of patients who have received cardiac transplants have had end-stage heart
disease due to coronary artery disease or cardiomyopathy

D. Patients with heart transplants have limited exercise capacity because of absence of
autonomic neural control

52
E. Endomyocardial biopsy is the most reliable technique to assess allograft rejection

146. All of the following statements regarding syncope are true EXCEPT:

A. The cause of syncope is not identified in up to 50 percent of patients

B. One-year mortality in patients with unknown cause of syncope is about 6 percent

C. One-year mortality in patients with syncope due to noncardiovascular causes is less than 15
percent

D. One-year mortality in patients with syncope due to cardiovascular cause is 19 to 30 percent

E. The diagnostic yield of an electrophysiological study (EPS) in patients with unexplained


syncope and structurally normal hearts is about 70 percent

147. All of the following statements about cardiac hypertrophy in athletes are true EXCEPT:

A. Isotonic exercise such as running and swimming increases LV end-diastolic diameter

B. Isometric exercise such as weight-lifting may cause an increase in LV end-diastolic diameter

C. Both isotonic and isometric exercise cause an increase in LV mass

D. Both isometric and isotonic exercise cause no change in LV wall thickness

E. Cardiac hypertrophy secondary to exercise usually disappears rapidly when training is


discontinued

148. Each of the following statements regarding cardiac transplantation is true EXCEPT:

A. Recipient age >55 years is an absolute contraindication to cardiac transplantation

B. Pulmonary hypertension, with a pulmonary vascular resistance >6 Wood units, is a


contraindication to receiving a cardiac transplant

C. A measured peak oxygen uptake ( O2max) of less than 10 ml/kg/min warrants consideration
of cardiac transplantation

D. Donor ischemic times of <6 hours are acceptable for donor hearts

E. The 1-year and 3-year survival rates of cardiac transplant recipients are 82 percent and 74
percent, respectively

53
149. A 78-year-old man who lives in a nursing home is admitted via the emergency room because of
fever and disorientation. His physical examination and ECG are normal except for sinus tachycardia
and tachypnea. Laboratory results include an elevated white blood cell count, low platelet count, and
prolonged prothrombin time. Urine sediment contains numerous polymorphonuclear leukocytes. A
cardiology consultation is obtained for evaluation of the chest x-ray in (Fig. 39). The most likely
explanation for the accompanying chest x-ray findings is:

A. LV failure

B. Pneumococcal pneumonia

C. Adult respiratory distress syndrome

D. Gram-negative pneumonia

E. Posterior wall MI

150. A pacemaker’s pulse generator is connected electrically to the heart via an electrode system
referred to as a lead. True statements with regard to unipolar and bipolar electrodes include all of the
following EXCEPT:

A. In bipolar systems both electrodes are located in the cardiac chamber

B. Pacing thresholds for generation of stimuli are similar for unipolar and bipolar electrodes

C. Unipolar electrodes are more susceptible to extracardiac interference from skeletal muscle
potentials (myopotentials)

D. The signal amplitudes of the electrograms generated by both types of electrodes are similar

E. Electromagnetic interference causes oversensing and pacemaker malfunction more frequently


with bipolar than with unipolar electrodes

151. The following statements about laboratory findings in heart failure are true EXCEPT:

A. Serum electrolyte values are usually normal in patients with untreated heart failure of short
duration

B. Hyponatremia in heart failure may be due to a combination of dietary sodium restriction,


diuretic therapy, and an elevated circulating vasopressin concentration

C. Elevated serum aspartate aminotransferase (AST) levels may accompany congestive


hepatomegaly due to heart failure

54
D. Acute hepatic venous congestion due to heart failure may produce a syndrome that closely
resembles viral hepatitis

E. Elevations of pulmonary capillary pressure to 13 to 17 mm Hg are commonly responsible for


pulmonary vascular redistribution and interstitial edema on the chest roentgenogram

152. Each of the following statements about myocardial contraction is true EXCEPT:

A. Beta1-adrenergic stimulation increases the concentration of intracellular calcium

B. Beta1-adrenergic stimulation promotes production of intracellular cyclic guanosine


monophosphate (cGMP)

C. Interaction of calcium with troponin-C is essential for myocyte contraction

D. Myosin molecules are tethered to the Z-line by the protein titin

E. The sarcoplasmic reticulum plays a key role in the release and uptake of calcium

153. All of the following interventions may lower blood pressure EXCEPT:

A. A diet that reduces caloric intake by 1000 calories per day

B. Reduction of dietary sodium

C. A scheduled daily regimen of isometric exercise

D. Supplemental dietary calcium

E. Reduction of ethanol consumption to less than 1 oz (30 ml)/d

154. Each of the following statements regarding cardiac transplant rejection is true EXCEPT:

A. Endomyocardial biopsy is the most reliable technique to assess rejection

B. Lymphocyte infiltration and myocyte necrosis are the most important biopsy features of post-
transplant rejection

C. Most episodes of acute rejection occur more than 3 months after transplantation

D. Total lymphoid irradiation has been used to treat patients with persistent or recurrent rejection

55
E. Pulsed corticosteroids are the therapy of choice for episodes of early rejection

155. Each of the following statements about procainamide is true EXCEPT:

A. The plasma concentration of procainamide required to suppress premature ventricular


contractions in patients after MI may be less than that required to prevent sustained ventricular
tachycardia

B. Procainamide, like quinidine, may accelerate the ventricular response in patients with atrial
fibrillation or flutter

C. Procainamide may block conduction in the accessory pathway of patients with the Wolff-
Parkinson-White syndrome

D. Toxic concentrations of procainamide may diminish myocardial performance

E. Procainamide is a preferred drug for the control of the tachyarrhythmias in patients with
bradycardia-tachycardia syndrome

156. All of the following are primary determinants of cardiac output EXCEPT:

A. Heart rate

B. LV preload

C. LV afterload

D. Myocardial contractile or inotropic state

E. Oxygen-carrying capacity of blood

157. Each of the following statements about exercise testing in the diagnosis of cardiac arrhythmias is
true EXCEPT:

A. Approximately one third of normal subjects develop ventricular ectopy during exercise testing

B. Nonsustained ventricular tachycardia of six beats or less can occur in normal subjects and
does not predict cardiovascular morbidity

C. Patients with ischemic heart disease develop premature ventricular contractions at lower heart
rates than normal subjects

D. Exercise testing should be avoided in patients with a history of serious ventricular arrhythmia

56
E. Exercise testing is less sensitive than prolonged ambulatory monitoring in detecting ventricular
ectopy

158. With respect to renovascular disease, all of the following statements are true EXCEPT:

A. Fewer than 2 percent of adults with hypertension in a general practice have renovascular
hypertension

B. Atherosclerotic disease most commonly involves the proximal third of the main renal artery

C. The most common form of fibroplastic renovascular disease in adults involves the media

D. The incidence of renovascular hypertension is higher in blacks than whites

E. Patients with severe, accelerated hypertension have the highest prevalence of renovascular
disease

159. Each of the following statements about invasive electrophysiology study of the cardiac
conduction system is true EXCEPT:

A. Ventricular tachyarrhythmia is a common explanation for syncope or presyncope in patients


with an intraventricular conduction disturbance

B. The presence of an His-ventricular (HV) interval equal to or greater than that recorded during
normal sinus rhythm is consistent with the diagnosis of a ventricular tachycardia

C. In patients with sinus node dysfunction, associated impaired AV conduction is commonly


found

D. Sinus node recovery time (SNRT) is defined as the difference between the spontaneous sinus
node cycle length prior to pacing and the duration of the first spontaneous sinus response after
termination of pacing

E. Accessory pathways are most commonly located in the LV free wall

160. True statements about the use of adenosine in the management of cardiac arrhythmia include all
of the following EXCEPT:

A. Adenosine is useful in the treatment of patients with supraventricular tachycardia

B. Intravenous bolus injections of adenosine at doses of 6 to 12 mg will terminate more than 90


percent of supraventricular tachycardias

57
C. Intravenous verapamil remains the agent of choice to terminate acutely a supraventricular
tachycardia

D. Adenosine is useful in the diagnosis of wide QRS tachycardias

E. Flushing, dyspnea, and chest pressure are all common side effects of adenosine therapy

161. Each of the following statements with respect to primary aldosteronism is true EXCEPT:

A. The most common cause for primary aldosteronism is a solitary benign tumor

B. Hypokalemia (<3.2 mmol/L) is present in the majority of patients

C. In patients with primary aldosteronism not receiving diuretics or supplemental potassium,


urinary potassium will be >30 mEq/d

D. Patients with primary aldosteronism usually have high plasma renin activity

E. If a patient has high urinary potassium and low serum potassium but low serum aldosterone
levels, licorice ingestion should be considered as a possible cause of the laboratory abnormalities

162. Comparison of LV parameters in pressure- and volume-overloaded hearts reveals all of the
following changes EXCEPT:

A. LV systolic stress initially is greater in pressure overload

B. Eccentric hypertrophy is characteristic of volume overload

C. Concentric hypertrophy is characteristic of pressure overload

D. LV wall thickness is greater in pressure overload

E. LV peak systolic pressure is greater in volume overload

163. All of the following are correct statements with respect to management of hypertension in
pregnancy EXCEPT:

A. Sodium restriction is an important component of therapy

B. Restriction of physical activity is advisable

C. Diuretics are usually contraindicated

58
D. Beta-blockers may be useful

E. Methyldopa is frequently prescribed

164. Each of the following statements regarding coronary artery disease in cardiac transplant
recipients is true EXCEPT:

A. The incidence of coronary artery disease is 20 to 50 percent at 5 years after transplantation

B. Histological features of graft arteriopathy include intimal proliferation with hyperplasia of


smooth muscle and lipid-laden macrophages

C. Immunosuppressive therapy with cyclosporine decreases the incidence of graft arteriopathy

D. Cytomegalovirus (CMV) infection may contribute to graft arteriopathy

E. Lipid lowering with HMG CoA reductase inhibitor therapy has been reported to decrease the
incidence of graft arteriopathy

165. Each of the following statements about the clinical manifestations of digitalis toxicity is correct
EXCEPT:

A. Digitalis overdose may lead to nausea and vomiting due to central nervous system
mechanisms

B. Digitialis intoxication may result in malaise, disorientation, seizures, or other neurological


symptoms

C. Digitalis may induce gynecomastia in men

D. Paroxysmal atrial tachycardia with AV block (PAT with block) is virtually pathognomonic of
digitalis excess

E. Common arrhythmias due to digitalis toxicity include atrioventricular junctional escape rhythms
and ventricular bigeminy or trigeminy

166. All of the following statements about shunt detection are true EXCEPT:

A. When “physiological” shunt is present, arterial oxygen saturation normalizes with


administration of 100 percent oxygen

B. Methods of shunt detection include oximetry, echocardiography, radionuclide, and magnetic


resonance imaging

59
C. Among the sources of right atrial venous blood, the inferior vena cava has the lowest oxygen
saturation

D. Although the sensitivity of oximetry for shunt detection is low, most clinically relevant left-to-
right shunts can be detected using this method

E. The Flamm formula is used to estimate mixed venous oxygen saturation

167. Each of the following statements about the use of digoxin in heart failure is correct EXCEPT:

A. Clinical trials have demonstrated that digoxin reduces mortality in patients with chronic heart
failure

B. The ideal trough serum digoxin level, for maximum hemodynamic benefit and safety, is 0.5 to
1 ng/dl

C. Hypothyroidism increases the likelihood of digoxin toxicity

D. Quinidine, verapamil, and propafenone can each increase the serum digoxin level

E. Both hypokalemia and hypercalcemia increase the risk of digoxin-associated arrhythmias

168. Each of the following statements about vasodilator agents used in heart failure is true EXCEPT:

A. The most important adverse effect of sodium nitroprusside is hypotension

B. Sublingual nitroglycerin may be used to effect a rapid reduction in LV filling pressures

C. Chronic administration of a combination of hydralazine and isosorbide dinitrate has been


shown to prolong survival in patients with heart failure

D. The combination of hydralazine plus isosorbide dinitrate reduces mortality in chronic heart
failure at least as much as ACE inhibitors.

E. ACE inhibitors lead to a decline in left and right ventricular filling pressures with little or no
change in heart rate in patients with heart failure

169. A 45-year-old woman with a history of breast cancer presents because of progressive shortness
of breath. Her ECG is shown in (Fig. 40). Each of the following statements regarding her condition is
true EXCEPT:

A. A third heart sound is absent

60
B. A prominent y-descent is expected in the right atrial pressure tracing

C. Kussmaul sign is not likely

D. An inspiratory decline of the systolic arterial pressure of >10 mm Hg is expected

E. Diuretic therapy should be avoided

170. All of the following statements concerning measurement of cardiac output are true EXCEPT:

A. Angiographic cardiac output measurements are preferred to Fick and thermodilution methods
for calculation of stenotic valve areas in patients with severe aortic or mitral regurgitation

B. There is a variability of up to 10 percent in calculations of cardiac output by the Fick method

C. When interpreting results of the indicator-dilution method, the area under the curve is directly
related to the cardiac output

D. Left-to-right shunts result in an early recirculation on indicator-dilution or thermodilution curves

E. Thermodilution cardiac outputs may cause a falsely elevated cardiac output value in low-
output states

171. Each of the following statements concerning therapy of patients with LV dysfunction is true
EXCEPT:

A. Use of amiodarone in patients with LV dysfunction has been shown to increase mortality

B. An implantable cardiac defibrillator (ICD) is the therapy of choice in patients with LV


dysfunction who have survived sudden death

C. An ICD is the therapy of choice in patients with LV dysfunction who have symptomatic
sustained ventricular tachycardia

D. An ICD is the therapy of choice in patients with LV dysfunction who have asymptomatic
nonsustained, but inducible ventricular tachycardia

E. Biventricular pacing increases LV ejection fraction, reduces wall stress, and decreases filling
pressures

172. Each of the following statements regarding antiarrhythmic therapy with quinidine is true
EXCEPT:

61
A. The most common adverse effects of quinidine are gastrointestinal

B. Torsades de pointes occurs in up to 3 percent of patients treated with quinidine

C. Quinidine has been associated with immune-mediated thrombocytopenia and hemolytic


anemia

D. Quinidine elevates the serum digoxin level

E. Monotherapy with quinidine is an appropriate treatment for atrial flutter

173. Each of the following statements concerning atrial fibrillation (AF) is true EXCEPT:

A. AF is commonly seen in patients following cardiac surgery

B. In patients with underlying cardiovascular disease, the development of chronic AF increases


overall mortality

C. Physical findings in patients with AF include variations in the intensity of S1 and in the
amplitude of a waves in the jugular venous pulse

D. A greater frequency of right bundle branch block is noted in patients who develop AF in the
year following acute MI

E. In the absence of underlying heart disease, subjects with AF have no increase in cardiac risk
but do display a higher incidence of stroke than the general population

174. Each of the following statements regarding hypertension is true EXCEPT:

A. Purely “white coat” hypertension is found in 20 to 30 percent of patients

B. When measuring the blood pressure, an inappropriately small cuff size results in a spuriously
low systolic measurement

C. Pseudo-hypertension may occur in patients with sclerotic brachial arteries

D. Chronic renal disease is the second most common cause of hypertension after essential
hypertension

E. Coarctation of the aorta, Cushing’s disease, primary aldosteronism, and pheochromocytoma


together account for approximately 1 percent of all hypertensive patients

62
175. Each of the following statements regarding the cardiac conduction system is true EXCEPT:

A. The sinus node is innervated with postganglionic adrenergic and cholinergic nerve terminals

B. In 60 percent of people, the arterial supply to the AV node is derived from a branch of the left
circumflex artery

C. The conduction system in the upper muscular interventricular septum receives its blood supply
from branches of the anterior and posterior descending arteries

D. Inhibition of the delayed rectifier K+ current (IKr) has been implicated in the acquired form of the
long QT syndrome

E. The resting transmembrane potential of the cardiac myocyte is close to the equilibrium
potential of potassium

176. True statements about the syndrome of circulatory shock include all of the following EXCEPT:

A. The clinical signs of shock reflect a decrease in blood flow to a variety of organs

B. ECG signs of myocardial ischemia may appear in patients with apparently normal hearts due
to a reduction in regional coronary blood flow

C. Circulatory shock in the first 3 months of life is often due to gram-negative bacteremia

D. Increases in capillary hydrostatic pressure during circulatory shock lead to a depletion of


plasma water and hemoconcentration

E. During circulatory shock, pulmonary blood flow is often protected at the expense of cerebral
and renal perfusion

177. Each of the following statements regarding amiodarone is true EXCEPT:

A. The mean terminal half-life of amiodarone is about 2 months

B. Use of amiodarone in patients with congestive heart failure increases the mortality rate

C. Corneal deposits develop in nearly all patients who are treated with amiodarone for more than
6 months

D. Amiodarone can cause either hypo- or hyperthyroidism

E. Amiodarone-induced pulmonary toxicity may develop within the first week of therapy

63
178. Each of the following statements about edema in heart failure is correct EXCEPT:

A. Edema in heart failure does not correlate well with the level of systemic venous pressure

B. Peripheral edema may be detected when extracellular fluid volume has increased by as little
as 1 to 2 L

C. Severe edema may cause rupture of the skin and extravasation of fluid

D. In patients with acute heart failure, edema may not be present initially

E. In patients with hemiplegia due to a cerebral vascular accident, edema is usually more
apparent on the paralyzed side

179. Major alterations in pulmonary function testing in patients with congestive heart failure include all
the following EXCEPT:

A. Vital capacity is reduced

B. Total lung capacity is reduced

C. Pulmonary compliance is reduced

D. Residual volume is reduced

E. Pulmonary diffusion capacity at rest is reduced

180. Each of the following statements regarding cardiac arrhythmias is true EXCEPT:

A. In the common form of AV nodal reentry tachycardia (AVNRT), anterograde conduction occurs
down the “slow” pathway

B. In antidromic AV reciprocating tachycardia, the activation wave travels via the accessory
pathway to the ventricles and retrogradely via the AV node to the atria

C. In most patients with Wolff-Parkinson-White syndrome, the accessory pathway conducts more
rapidly than the normal AV node but takes longer to recover excitability

D. Abnormal automaticity is the most common mechanism of AF

E. Ventricular tachycardia can be caused by re-entry, triggered activity, or abnormal automaticity

64
181. Each of the following statements about high-output heart failure is true EXCEPT:

A. Thyrotoxicosis, anemia, and pregnancy are all examples of high-output states that may lead to
heart failure

B. The extremities of the patient with high-output failure are usually warm and flushed

C. Vasodilator therapy is usually helpful in heart failure due to anemia

D. The pulse pressure in high-output failure is normal or widened

E. Evidence of biventricular failure, sensory and motor peripheral neuropathy, and prolongation of
the QT interval is consistent with the diagnosis of thiamine deficiency (beriberi)

182. Each of the following statements about verapamil is true EXCEPT:

A. Verapamil exerts a direct myocardial depressant effect

B. Cardiac index may remain unchanged in patients on verapamil because afterload reduction
produced by the drug counteracts its negative inotropic effect

C. Verapamil is helpful in slowing the rapid ventricular response in patients with atrial fibrillation
and wide QRS complexes due to Wolff-Parkinson-White syndrome

D. Verapamil is a useful drug for terminating sustained AVNRTs

E. Verapamil has some alpha-adrenergic blocking and antiplatelet effects

183. Each of the following statements about physical findings in heart failure is true EXCEPT:

A. Chronic marked elevation of systemic venous pressure may produce exophthalmos or even
visible systolic pulsation of the eyes

B. Pallor and coldness of the extremities are primarily due to elevation of adrenergic nervous
system toxicity

C. A positive abdominojugular reflex reflects the combination of hepatic congestion and the
inability of the right side of the heart to accept an increased venous return

D. The presence of hepatic tenderness reflects long-standing right-sided heart failure with chronic
stretching of the liver capsule

E. Protein-losing enteropathy may occur in patients with visceral congestion and may result in a
reduced plasma oncotic pressure

65
184. Each of the following statements regarding class III antiarrhythmic drugs is true EXCEPT:

A. Sotalol results in torsades de pointes in up to 4 percent of patients

B. Sotalol can precipitate heart failure in patients with reduced baseline systolic function

C. After ibutilide administration, patients can be safely discharged 2 hours after conversion from
atrial flutter to normal sinus rhythm

D. Ibutilide results in conversion to sinus rhythm in up to 60 percent of patients with atrial


fibrillation and 70 percent with atrial flutter

E. Ibutilide therapy should be avoided in patients with QTc interval of >440 msec

DIRECTIONS: Each question below contains suggested answers. For EACH of the alternatives, you
are to respond either TRUE (T) of FALSE (F). In a given item, ALL, SOME, OR NONE OF THE
ALTERNATIVES MAY BE CORRECT.

185. Digitalis is of potential benefit in which of the following conditions? For the following choices,
answer true (T) or false (F).

A. Mitral stenosis with atrial fibrillation and normal right ventricular function

B. LV hypertrophy, normal sinus rhythm, normal LV ejection fraction, and elevated LV end-diastolic
pressure

C. Hypertrophic obstructive cardiomyopathy with LV ejection fraction of 70 percent and normal sinus
rhythm

D. Mitral stenosis with normal sinus rhythm and normal right ventricular function

E. Hypertrophic obstructive cardiomyopathy with LV ejection fraction of 70 percent and atrial


fibrillation

186. For each of the following statements regarding the cardiac cycle, you are to respond true (T) or
false (F).

A. The third heart sound (S3) corresponds to the rapid early diastolic filling of the ventricles

B. A prominent a wave on the right or left atrial pressure tracing is typical of atrial fibrillation

C. The QRS complex on the ECG corresponds to the timing of isovolumic ventricular contraction

D. The v wave on the right or left atrial pressure tracing occurs after the T-wave on the ECG

E. The mitral valve opens in diastole when the left atrial pressure falls below that of the left ventricle

66
187. Common features in the clinical presentation of renovascular hypertension secondary to
fibromuscular hyperplasia, as opposed to atherosclerosis, include the following:

A. Age <50 years

B. Female gender

C. Coexisting cardiomegaly

D. No family history of hypertension

E. Presence of carotid bruits

188. Indicate whether the following statements about physical findings in heart failure are true (T) or
false (F):

A. Hydrothorax in heart failure is most often bilateral, but when unilateral it is usually confined to the
left side of the chest

B. The absence of pulmonary rales on examination excludes the presence of an elevated pulmonary
capillary pressure

C. Hepatomegaly frequently precedes the development of overt peripheral edema

D. The absence of peripheral edema is a reliable indicator that the systemic venous pressure is
normal

E. In LV failure, P2 is often accentuated

188. Indicate whether the following statements about physical findings in heart failure are true (T) or
false (F):

A. Hydrothorax in heart failure is most often bilateral, but when unilateral it is usually confined to the
left side of the chest

B. The absence of pulmonary rales on examination excludes the presence of an elevated pulmonary
capillary pressure

C. Hepatomegaly frequently precedes the development of overt peripheral edema

D. The absence of peripheral edema is a reliable indicator that the systemic venous pressure is
normal

E. In LV failure, P2 is often accentuated

189. For each of the conditions listed below respond true (T) if the condition is associated with high-
output heart failure and false (F) if it is not.

A. Iron overload

B. Hyperthyroidism

67
C. Systemic arteriovenous fistulas

D. Thiamine deficiency

E. Paget’s diseas

190. For each of the suggested answers regarding treatment of digitalis intoxication, you are to
respond either true (T) or false (F).

A. Lidocaine and phenytoin are useful agents in treating arrhythmias due to digitalis excess

B. Second- and third-degree AV block often respond to atropine

C. Recurrence of digitalis toxicity can occur 24 to 48 hours following the administration of antidigoxin
Fab antibodies

D. Dialysis is very helpful in cases of massive overdose

E. Direct-current cardioversion is safe in cases of digitalis-induced ventricular arrhythmias

191. For each of the following statements about heart failure, respond either true (T) or false (F):

A. Over the past decade, the incidence and prevalence of heart failure have increased

B. Heart failure occurs in 10 percent of patients over 75 years old but only in 1 to 2 percent of patients
50 to 59 years of age

C. Orthopnea is a specific symptom of heart failure

D. Pulsus alternans occurs most commonly in diastolic heart failure

192. True statements regarding hypomagnesemia in patients with congestive heart failure include
which of the following?

A. Both thiazide and loop diuretics may cause urinary magnesium wasting

B. Chronic exposure to ethyl alcohol leads to increased renal magnesium losses

C. Digitalis may lead to renal magnesium wasting

D. A total daily urinary magnesium excretion of <1 mEq in the absence of diuretics is suggestive of
magnesium depletion

E. Elevated magnesium levels in congestive heart failure may predict as poor a prognosis as
hypomagnesemia

193. For each of the following statements regarding the use of adenosine as an antiarrhythmic agent,
respond either true (T) or false (F):

A. The elimination half-life of adenosine is less than 6 seconds

B. Theophylline inhibits adenosine’s antiarrhythmic effect

68
C. The dose of adenosine should be doubled in patients taking dipyridamole

D. Adenosine is contraindicated in wide-complex tachycardias

E. Flushing, dyspnea, and chest pain develop in up to 40 percent of patients after adenosine
administration

194. A patient presents with a wide-complex tachycardia. Which of the following statements are true
in distinguishing ventricular tachycardia (VT) from supraventricular tachycardia (SVT) with aberrant
ventricular conduction?

A. The presence of fusion beats supports the diagnosis of VT

B. Termination of the tachycardia by vagal maneuvers is consistent with SVT

C. The presence of an RsR pattern in lead V1 is more suggestive of SVT

D. AV dissociation implies the presence of VT

E. A past history of MI makes the diagnosis of VT more likely

195. For each of the following statements regarding electrical cardioversion, respond either true (T) or
false (F):

A. Electrical cardioversion is the treatment of choice for tachyarrhythmias caused by digitalis toxicity

B. Synchronized cardioversion should be used when electrically converting SVTs

C. The incidence of systemic embolism following successful cardioversion of atrial fibrillation is 1 to 3


percent

D. Repeated shocks at the same energy level increase chest wall impedance

E. Anticoagulation therapy can be safely discontinued 5 days following successful cardioversion of


chronic atrial fibrillation to normal sinus rhythm

196. For each of the following statements regarding the antiarrhythmic drug disopyramide, respond
either true (T) or false (F):

A. Disopyramide should be avoided in patients with glaucoma

B. Disopyramide may increase the sinus node discharge rate because of its muscarinic receptor
blocking activity

C. The elimination half-life is prolonged in renal or hepatic failure

D. It is poorly tolerated by patients with heart failure

E. It can cause QT prolongation and result in torsades de pointes

69
197. AV block exists when the atrial impulse is conducted with delay or is not conducted at all to the
ventricle. True statements about second-degree AV block include which of the following?

A. Type I AV block is associated with a benign clinical course in all age groups

B. Type II AV block in inferior MI is not associated with an increase in mortality

C. Type I AV block with a normal QRS complex occurs at the level of the AV node or in the His bundle

D. Type II AV block in association with bundle branch block occurs at the level of the AV node or the
His bundle

E. 2:1 AV block may be either type I or type II second-degree AV block and is more likely to be type I
if the QRS complex is normal

198. For each of the following conditions, respond either true (T) or false (F). LV preload is increased
in the presence of:

A. Sepsis

B. Right ventricular infarct

C. Mitral regurgitation

D. Dehydration

E. Renal failure with oliguria

199. A 74-year-old man with a long history of LV failure secondary to several MIs comes to the
emergency room acutely short of breath 2 hours after eating a large holiday meal. Physical
examination and chest x-ray findings are consistent with acute pulmonary edema. ECG shows a
narrow-complex junctional tachycardia at a rate of 130/min with 1 mm ST segment depression in
leads V4 through V6. Blood pressure is 170/100; respirations, 32. His current medical treatment
includes nitrates, calcium channel antagonists, digoxin, and chlorothiazide. Appropriate initial therapy
in the emergency room would include:

A. Nasal O2

B. Morphine sulfate

C. Intravenous furosemide

D. Intravenous digoxin

E. Sublingual nitroglycerin

70
200. Permanent cardiac pacing is appropriate for which of the following patients with AV block?
Respond either true (T) or false (F):

A. A 35-year-old man with asymptomatic type I second-degree AV block

B. A 70-year-old man with LV hypertrophy, persistent fatigue, and lightheadedness, with marked first-
degree AV block (PR interval = 0.36 sec)

C. A 57-year-old man with acquired asymptomatic third-degree AV block

D. A 40-year-old woman with symptomatic congenital complete AV block

E. A 56-year-old man with intermittent type II second-degree AV block

201. A 60-year-old man had a pacemaker inserted 5 years ago because of sinus bradycardia with
poor LV function following an inferior MI. As part of his exercise program, he has been using a rowing
machine. Recently, he has had several episodes of near-syncope that occurred only while rowing. An
ambulatory ECG (Holter monitor) revealed an abnormal rhythm during a near-syncopal event while
rowing. The ECG shown in (Fig. 41) indicates:

A. There is evidence of a dual-chamber pacing system

B. There is inappropriate inhibition of ventricular pacing

C. There is undersensing of atrial activity

D. There is lack of capture of the ventricles

E. There is lack of capture of the atria

202. True statements about digoxin include:

A. The half-life of digoxin in subjects with normal renal function is about 36 to 48 hours

B. In patients with normal renal function who have not previously taken digitalis, institution of daily
maintenance therapy without a loading dose leads to a steady-state plateau concentration of the drug
in about 7 days

C. Dialysis is a relatively effective way of removing digoxin from the body

71
D. Obese patients taking digoxin who undergo dramatic weight loss must have their digoxin dosage
decreased to avoid toxicity

E. Patients taking digoxin who subsequently begin propafenone therapy often require a decrease in
digoxin dosage to avoid toxicity

203. Each of the following conditions is associated with the development of pulmonary edema
EXCEPT:

A. Increased pulmonary venous pressure

B. High altitude

C. Increased plasma oncotic pressure

D. Eclampsia

E. Heroin overdose

204. For each of the suggested answers regarding electrical therapy of cardiac arrhythmias, respond
either true (T) or false (F):

A. Direct-current cardioversion is successful in terminating up to 95 percent of reentrant arrhythmias

B. Direct-current cardioversion can be safely used to terminate digitalis-induced reentrant arrhythmias

C. Synchronized shocks delivered late in the QRS complex are more effective and less likely to
induce acceleration of the arrhythmia than shocks delivered near the QRS onset

D. When properly synchronized, direct-current shock does not result in ventricular fibrillation

E. Elevation of creatine kinase-MB fraction following cardioversion is common

205. For each of the following statements regarding hypertension, indicate whether the statement is
true (T) or false (F):

A. Increased LV muscle mass is a strong and independent risk factor of cardiac mortality

B. The risk of ventricular arrhythmias is increased at least twofold in the presence of LV hypertrophy

C. Low birth weight is associated with an increased incidence of hypertension later in life

D. Women have greater cardiovascular morbidity and mortality than men for all degrees of
hypertension

E. In hypertensive adolescents, essential hypertension is the most likely diagnosis

206. True statements about cardiac physical findings in patients with congestive heart failure include:

A. Heart failure due to restrictive cardiomyopathy is usually accompanied by signs of cardiomegaly

B. Pulsus alternans occurs most commonly in heart failure due to concomitant mitral regurgitation

72
C. Pulsus alternans results from variation of the stroke volume due to incomplete recovery of
contracting myocardial cells

D. A low-grade fever that is due to cutaneous vasoconstriction and impaired heat loss may occur in
severe heart failure

E. Cheyne-Stokes respiration is a cyclic respiratory pattern that results from a combination of a


change in the sensitivity of the respiratory center and the presence of LV failure

207. Correct statements regarding the association of oral contraceptive pills and hypertension include:

A. The likelihood of developing hypertension is increased by significant alcohol consumption

B. The incidence of hypertension is about 2.5 times greater in pill users than in nonusers

C. The likelihood of developing hypertension is unaffected by the age of the user

D. The mechanism for contraceptive-induced hypertension probably involves renin-aldosterone-


mediated volume expansion

E. Cigarette smoking more than doubles the cardiovascular complications associated with use of oral
contraceptives

208. Which of the following statements regarding sinus node function are true? Respond either true
(T) or false (F).

A. Sinus bradycardia is a common rhythm in well-trained athletes

B. During sleep, the heart rate of normal individuals can fall to 35 beats/min

C. Sinus arrest is identified as a pause that is an exact multiple of the P-P interval of the underlying
rhythm

D. Sinus arrest and AV block are common in patients with sleep apnea

E. In the respiratory form of sinus arrhythmia, the P-P interval cyclically shortens during inspiration

209. True statements with regard to distribution of blood flow and intravascular pressure in the upright
lung include:

A. Pulmonary artery pressure is greater than alveolar pressure at the lung apices

B. Pulmonary venous pressure exceeds alveolar pressure at the lung bases

C. Alveolar pressure increases from the lung base to the lung apex

D. Pulmonary vascular redistribution occurs when there is a relative reduction in perfusion of the
bases with a relative increase in apical perfusion

210. Which of the following are considered complications of cyclosporine therapy in the cardiac
transplant recipient?

A. Renal dysfunction

73
B. Bone marrow suppression

C. Hepatotoxicity

D. Neurological effects

E. Hirsutism

211. For each of the following statements regarding pheochromocytoma, indicate if the statement is
true (T) or false (F).

A. Approximately 15 percent of pheochromocytomas are extra-adrenal

B. Ten percent of pheochromocytomas are malignant

C. Ten percent of adrenal pheochromocytomas are bilateral

D. Urinary metanephrine levels increase after the administration of x-ray contrast media containing
methylglucamine

E. Pheochromocytoma is inherited in some cases

212. For each of the following statements regarding arrhythmogenic right ventricular dysplasia
(ARVD), respond either true (T) or false (F):

A. Ventricular tachycardia in patients with ARVD typically has a right bundle branch block morphology

B. ARVD is more common in women

C. Fatty or fibrofatty infiltration of the right ventricle is the pathological hallmark

D. Pathological changes do not occur in the left ventricle

E. Radiofrequency ablation is successful at preventing ventricular tachycardia

213. Features of the systemic lupus erythematosus (SLE)-like syndrome that may be induced by
procainamide include:

A. The development of antinuclear antibodies in 20 to 30 percent of patients on chronic procainamide


therapy

B. The development of clinical symptoms of SLE in 20 to 30 percent of patients on chronic


procainamide therapy

C. A similar occurrence of SLE syndrome due to the N-acetylprocainamide (NAPA) metabolite of the
drug

D. A response to steroids in some patients

74
214. For each of the following statements regarding the hypersensitive carotid sinus syndrome,
respond either true (T) or false (F):

A. Atropine abolishes the cardioinhibitory form of this syndrome

B. During carotid sinus stimulation, a decrease in systolic blood pressure >30 mm Hg and
reproduction of the patient’s symptoms is consistent with the vasodepressor form of this syndrome

C. The hypersensitive carotid sinus reflex is commonly associated with coronary artery disease

D. Clonidine is a frequently beneficial treatment of carotid sinus hypersensitivity

E. Single-chamber atrial pacing is sufficient therapy for most patients with carotid sinus
hypersensitivity

215. For each of the following statements regarding the antiarrhythmic drug amiodarone, respond
either true (T) or false (F):

A. Amiodarone has peripheral and coronary vasodilator properties

B. Corneal microdeposits develop in fewer than 10 percent of patients on long-term therapy

C. Amiodarone is readily removed by dialysis

D. In patients with heart failure symptoms due to dilated cardiomyopathy, the addition of amiodarone
to standard therapy has been shown to reduce mortality

216. A 62-year-old previously healthy man is brought to the emergency room because of severe
headache, nausea, and dizziness. He takes no medications. The blood pressure is 176/98 mm Hg;
heart rate is 56 and regular. The presenting ECG is as shown in (Fig. 42). Which of the following
actions are appropriate? Respond true (T) or false (F).

A. Initiate antiplatelet therapy with aspirin and a glycoprotein IIb/IIIa antagonist

B. Initiate antithrombotic therapy with heparin

C. Initiate anti-ischemic therapy with intravenous nitroglycerin and a beta-blocker

75
D. Obtain a head CT scan

E. Proceed directly to cardiac catheterization if ST-T wave abnormalities fail to quickly normalize with
anti-ischemic therapy

217. Indicate whether each of the following statements regarding sudden cardiac death (SCD) is true
(T) or false (F):

A. There are approximately 300,000 SCDs annually in the United States

B. The peak incidence of SCD is between the ages of 30 and 45 years

C. Hereditary causes of SCD include hypertrophic cardiomyopathy, long QT syndrome, right


ventricular dysplasia, and the Brugada syndrome

D. SCD is more common in women

E. An intraventricular conduction abnormality on the ECG is a stronger predictor of SCD than findings
of LV hypertrophy

218. A patient with the ECG shown in (Fig. 43) could also exhibit:

A. A gradual increase in atrial rate with the administration of digitalis

B. An irregular atrial rate

C. Precipitation of the arrhythmia by hypokalemia

D. An absence of underlying cardiac disease in 50 percent of cases

219. For each of the following statements regarding SCD, respond either true (T) or false (F):

A. SCD is the first clinical manifestation of coronary artery disease in approximately 25 percent of
patients

B. LV dysfunction and ventricular ectopic activity after an MI increase the risk of SCD

C. The most common mechanism of cardiac arrest is asystole

D. Onset of brain death occurs within 4 to 6 minutes of unattended ventricular fibrillation

E. The outcome of patients with bradycardic/asystolic out-of-hospital cardiac arrest is worse than if
ventricular fibrillation is the initial arrhythmia

76
220. Which of the following statements regarding electrophysiological testing are true?

A. A long His-ventricular (HV) interval (>80 msec) identifies patients at increased risk of developing
AV block

B. The HV interval has a high sensitivity and low specificity for predicting the development of
complete AV block

C. A prolonged sinus node recovery time (SNRT) is very specific for identifying patients with sinus
node dysfunction

D. The sensitivity of the SNRT in identifying sinus node dysfunction is about 95 percent

E. The complication rate of electrophysiological testing is 1 to 3 percent

221. Lidocaine is a widely used pharmacological agent for the treatment of cardiac arrhythmias.
Correct statements about lidocaine include:

A. Lidocaine has little effect on the electrophysiological properties of atrial myocardial cells or on
conduction in accessory pathways

B. In the absence of severe LV dysfunction, clinically significant adverse hemodynamic effects from
lidocaine are rarely noted

C. The elimination half-life of lidocaine in patients after relatively uncomplicated MI is two to four times
that in normal subjects

D. Patients treated with an initial bolus of lidocaine followed by a maintenance infusion may
experience transient excessive plasma concentrations of the drug 30 to 120 minutes after therapy is
begun

222. For each of the following statements regarding specific modes of cardiac pacing respond true (T)
or false (F):

A. Ventricular Inhibited Pacing (VVI) restores and maintains AV synchrony

B. VVI pacing provides rate responsiveness in the chronotropically incompetent patient

C. Single-chamber triggered pacing (AAT or VVT) increases the drain on the pacemaker battery

D. Atrial Inhibited Pacing (AAI) is an appropriate mode of pacing for patients with AV nodal
dysfunction

E. Dual-Chamber Pacing and Sensing with Inhibition and Tracking (DDD) is the preferred mode of
pacing for patients in atrial fibrillation

223. For each of the following statements regarding cardiac pacemakers, respond either true (T) or
false (F):

A. Hyperkalemia can result in pacing and sensing threshold abnormalities

B. Lead dislodgement or inadequate initial lead placement should be suspected if true undersensing
is present

77
C. Industrial-strength welding equipment (more than 500 A) can cause significant pacemaker
interference

D. Right bundle branch block is an appropriate ECG pattern after successful transvenous pacemaker
implantation

E. Early pacemaker infections are most often caused by Staphylococcus aureus

224. For each of the following statements about patients with symptomatic heart failure, respond
either true (T) or false (F):

A. The plasma norepinephrine level is usually elevated

B. Atrial natriuretic peptide is released in response to increased atrial distension

C. A marked increase in cardiac beta-adrenergic receptor density is present

D. Activation of the renin-angiotensin system is a common neurohormonal compensatory mechanism

E. The circulating level of the inflammatory cytokine, tumor necrosis factor-alpha, is increased

225. For the following statements regarding post-cardiac transplant complications, respond either true
(T) or false (F):

A. Infectious complications are the most common cause of death after transplantation

B. CMV infection occurs commonly in the first month after transplantation

C. Prophylactic therapy with trimethoprim-sulfa-methoxazole prevents CMV infection

D. CMV-negative recipients who receive an allograft from a CMV-positive donor are the most
vulnerable for developing CMV infection

E. Transplant recipients have a threefold increase in the incidence of cancer compared with age-
matched controls

226. Which of the following statements regarding intra-aortic balloon (IAB) counterpulsation are true?

A. Patients with cardiogenic shock or mechanical complications of an acute MI often benefit from the
placement of an IAB

B. Aortic valve insufficiency is a strict contraindication to the use of an IAB

C. The IAB should be timed to deflate during the isovolumetric phase of LV contraction

D. The tip of the IAB should be positioned just distal to the left subclavian artery

E. Inflation of the IAB should be timed with atrial contraction

78
227. For the following statements regarding pacemaker complications, respond either true (T) or false
(F):

A. Acceptable rates of lead dislodgement are less than 1 percent for ventricular leads and less than 2
to 3 percent for atrial leads

B. In the presence of a wire fracture, the lead impedance is high

C. In the presence of an insulation break, the lead impedance is low

D. In the presence of lead dislodgement, the lead impedance is normal

E. Early infection after pacemaker implantation is most commonly caused by streptococci

228. In patients with marked LV dysfunction, which of the following statements is/are true regarding
pulsus alternans?

A. It is usually associated with electrical alternans

B. It is more readily detected in the femoral as compared to radial arteries

C. It can be appreciated with sphygmomanometry

D. It can be elicited by the assumption of erect posture

E. The strong and weak beats are almost always concordant on both sides of the circulation (i.e., both
ventricles)

229. For the following statements regarding side effects of antihypertensive agents, respond either
true (T) or false (F):

A. Pericardial effusions have been reported with the use of minoxidil

B. Flushing and tachycardia may result from hydralazine

C. Fewer than 3 percent of patients taking an ACE inhibitor will develop a cough during prolonged use

D. ACE inhibitor–induced cough may persist for 3 weeks or longer after discontinuation of the drug

E. A pure beta-blocker is the pharmacological therapy of choice for patients with pheochromocytoma

230. A 55-year-old previously healthy man is brought to the emergency room because of left-sided
chest pain over the past 3 hours. He denies shortness of breath or cough. The discomfort seems less
intense when he sits forward. The chest radiograph is unremarkable, and the ECG is shown in (Fig.
44). Which of the following statements regarding this patient’s condition are true?

79
A. Thrombolytic therapy is indicated if cardiac catheterization is not immediately available

B. Nitrates will substantially help relieve the chest pain

C. Aspirin should be administered

D. Glucocorticoids are appropriate as primary therapy

E. Cardiac isoenzymes can quickly differentiate the cause of the chest pain

231. For each of the following statements regarding hypertension during pregnancy, indicate if the
statement is true (T) or false (F):

A. Gestational hypertension is more common in primigravid women compared to those with prior
pregnancies

B. Gestational hypertension frequently recurs in subsequent pregnancies

C. Preeclampsia is more common in younger compared to older women

D. ACE inhibitors are a mainstay of antihypertensive therapy during pregnancy

E. Low doses of aspirin prevent preeclampsia

232. Clinical features of the Wolff-Parkinson-White (WPW) syndrome include:

A. An absence of underlying heart disease in most adults

B. A decrease in the frequency of paroxysmal tachycardia with increasing age

C. An association with Ebstein’s anomaly

D. The majority of tachycardias presenting as atrial fibrillation

80
233. True statements regarding the antiarrhythmic agent procainamide include:

A. Procainamide’s major metabolite, NAPA, is excreted almost exclusively via the kidneys

B. Marked hypotension is a potential side effect of intravenous administration

C. Procainamide should be avoided in patients with WPW syndrome with rapid atrial fibrillation

D. The serum digoxin level increases when procainamide is administered concurrently

E. During chronic procainamide therapy, fewer than 10 percent of patients develop antinuclear
antibodies

234. For each of the following statements regarding therapy of patients with heart failure, respond
either true (T) or false (F):

A. In asymptomatic patients with LV dysfunction, the use of ACE inhibitors decreases mortality

B. ACE inhibitors are indicated in patients with heart failure and LV dysfunction irrespective of the
functional class

C. Digoxin has been shown to decrease heart failure hospitalizations but has no effect on mortality

D. Spironolactone has been shown to decrease mortality in patients with New York Heart Association
(NYHA) class III-IV symptoms

E. Patients with heart failure should be instructed to avoid exercise

235. For each of the following statements regarding WPW syndrome, respond either true (T) false (F):

A. A positive delta wave deflection in lead V1 indicates that the accessory pathway is located in the
right ventricle

B. The most common location of accessory pathways is the lateral LV wall

C. ST and T wave abnormalities are expected

D. The majority of patients with WPW have otherwise normal hearts

E. The prevalence of WPW is higher in men and decreases with age

236. For each of the suggested answers, respond either true (T) or false (F). Thiazide diuretics may
cause many side effects, including:

A. Hypomagnesemia

B. Hypouricemia

C. Hypercalcemia

D. Hypercholesterolemia

E. Hyponatremia

81
DIRECTIONS: The questions below consist of lettered headings followed by a set of numbered items.
For each numbered item select the ONE lettered heading with which it is MOST closely associated.
Each lettered heading may be used ONCE, MORE THAN ONCE, or NOT AT ALL.

For each of the following descriptions, match the appropriate disorders:

A. Jervell and Lange-Nielsen syndrome

B. Romano-Ward syndrome

C. Right ventricular (RV) outflow tract ventricular tachycardia

D. Brugada syndrome

237. Familial disorder associated with sensorineural deafness

238. Typically, ECG reveals a right bundle branch block morphology with ST segment elevation in the
anterior precordial leads

239. Autosomal dominant long QT syndrome with normal hearing

240. Left bundle branch block with an inferior axis

For each condition, match the appropriate pattern of LV filling recorded by diastolic Doppler
mitral flow velocities (E wave = early diastolic filling; A wave = period of atrial contraction):

A. E wave > A wave, LV deceleration time > 190 msec

B. E wave > A wave, LV deceleration time < 190 msec

C. E wave < A wave, LV deceleration time > 200 msec

D. E wave >> A wave, LV deceleration time < 150 msec

241. Restrictive cardiomyopathy

242. Normal pattern

243. Pseudonormalized pattern

244. Impaired LV diastolic relaxation

Match the following antiarrhythmic drug actions with the appropriate Vaughan Williams drug
classification:

A. Predominantly block potassium channels and prolong repolarization

B. Predominantly block beta-adrenergic receptors

82
C. Predominantly block the slow calcium channels

D. Reduce the rate of rise of the action potential upstroke (Vmax) and prolong the action potential
duration

E. Block sodium channels, but shorten the action potential duration and do not reduce Vmax

245. Class IA drugs

246. Class II drugs

247. Class III drugs

248. Class IV drugs

For each condition capable of precipitating high-output cardiac failure, match the appropriate
phrase:

A. Hyperthyroidism

B. Beriberi

C. Arteriovenous fistula

D. Carcinoid syndrome

E. Osler-Weber-Rendu syndrome

249. Branham’s sign

250. Hepatomegaly and abdominal bruits

251. Means-Lerman scratch

252. Paresthesias and painful glossitis

For each disease state match the appropriate LV volume data.

End-Diastolic Volume (ml/m2) Stroke Volume (ml/m2) Mass (g/m2)


A. 70 45 92
B. 84 44 172
C. 193 92 200
D. 199 37 145
E. 70 40 80

83
253. Aortic valve stenosis with peak systolic gradient >30 mm Hg

254. Myocardial disease (primary dilated cardiomyopathy)

255. Aortic regurgitation with regurgitant flow >30 ml per beat

256. Mitral valve regurgitation with regurgitant flow >20 ml per beat

For each clinical condition, match the most appropriate pacemaker modality:

A. VAT

B. VVIR

C. DDD

D. DDDR

E. AAIR

257. A 58-year-old man with tachycardia-bradycardia syndrome who develops symptomatic sinus
bradycardia with beta-blocker therapy (which was given for inappropriate sinus tachycardia)

258. A 70-year-old woman with atrial fibrillation who complains of dizziness, and is found on
examination to have a ventricular rate of 30/min

259. A 62-year-old man with complete heart block following aortic valve surgery

260. A 45-year-old man with symptomatic sinoatrial exit block and junctional escape rhythm

For each description, match the appropriate cardiac rhythm:

A. Atrial tachycardia

B. Atrial flutter

C. Sinus rhythm

D. Atrioventricular nodal reentrant tachycardia (AVNRT)

E. Atrial fibrillation

261. P waves are negative in lead aVR and upright in leads I, II, aVF

262. Rhythm can be due to automaticity, reentry or triggered mechanisms

263. Macroreentrant mechanism in the right atrium

264. The initial P wave of the tachycardia is usually different than the subsequent P waves

84
Match each drug below with the potential adverse reactions:

A. Fever and agranulocytosis

B. Urinary retention

C. Seizures

D. Gingival hypertrophy

265. Disopyramide

266. Procainamide

267. Phenytoin

268. Lidocaine

For each of the following conditions, select the most appropriate statement about the physical
examination:

A. Absence of a waves in the jugular venous pulsation

B. Cannon a waves in the jugular venous pulsation

C. Paradoxical splitting of S2

D. Widened splitting of S2

269. Left bundle branch block

270. Right bundle branch block

271. Atrial fibrillation

272. Ventricular tachycardia

For each of the following conditions, match the corresponding clinical presentations of
syncope:

A. A 20-year-old woman “blacked out” during phlebotomy for a routine blood test

B. A 65-year-old woman lost consciousness following arm exercises

C. A 35-year-old man sustained syncope during exercise and has a systolic murmur that intensifies
upon standing upright

D. A 74-year-old man experiences sudden syncope while shaving

85
E. A 28-year-old woman with recurrent episodes of breathlessness, lightheadedness, and syncope
following changes in body position

273. Hypertrophic cardiomyopathy

274. Subclavian steal syndrome

275. Vasovagal syncope

276. Carotid sinus hypersensitivity

277. Left atrial myxoma

For each of the following antiarrhythmic agents, match the appropriate influence of disease
states on drug pharmacokinetics. Each lettered heading can be used ONCE, MORE THAN
ONCE, OR NOT AT ALL:

A. Lidocaine

B. Procainamide

C. Both

D. Neither

278. Reduced clearance of drug in the presence of liver disease

279. Reduced clearance of drug in the presence of kidney disease

280. Reduced clearance of drug in the presence of congestive heart failure

281. Reduced clearance of drug with prolonged infusion

For each of the following descriptions, match the appropriate medication:

A. One of the least lipid soluble beta-blockers

B. Cardioselective beta-blocker with intrinsic sympathomimetic activity

C. Beta-blocker with alpha-blocking activity

D. Noncardioselective beta-blocker with intrinsic sympathomimetic activity

282. Atenolol

283. Labetalol

284. Acebutolol

285. Pindolol

86
DIRECTIONS: Each question below contains five suggested responses. Select the ONE
BEST response to each question.

286. Which of the following is true regarding atrial septal defect (ASD) in children:

287. An 80-year-old man presents with syncope. During his evaluation, a cardiac murmur is detected
and an echocardiographic study obtained. A continuous-wave Doppler recording through the aortic
valve is shown in (Fig. 48). True statements about this case include all of the following EXCEPT:

A. Dyspnea is a more common presenting complaint than syncope in this disorder

B. Some patients with this disorder and angina will not have significant coronary arterial
obstruction on angiographic examination

C. Syncope in this disorder commonly occurs without significant change in systemic vascular
tone

D. Syncope may also have been due to an arrhythmia in this patient

E. Gastrointestinal bleeding may be associated with this disorder

288. A 45-year-old man comes to the office because of recurrent chest discomfort and shortness of
breath. He rides his bicycle daily to and from work and this past winter noticed occasional episodes of
discomfort during bicycling. An exercise tolerance test (ETT) was performed for 12 minutes with a
standard Bruce protocol without the development of significant symptoms or ECG findings. He was
placed on a beta-blocker, but his symptoms have persisted. You perform a repeat ETT, which again is

87
negative, except for upsloping ST-segment depressions present at 85 percent of maximal predicted
heart rate. You would recommend next:

A. Switching medications to a long-acting nitrate

B. No further work-up

C. Echocardiography

D. Stress test with nuclear scintigraphy

E. Coronary arteriography

289. Each of the following statements regarding the effects of HMG-CoA reductase inhibitors
(“statins”) is true EXCEPT:

A. The rate of cholesterol formation is decreased

B. Expression of hepatic low-density lipoprotein (LDL) receptors is increased

C. Hepatic production of very low-density lipoprotein (VLDL) is decreased

D. Reversible hepatotoxicity develops in approximately 10 percent of patients

E. Concurrent administration of erythromycin increases the risk of myositis

290. A 44-year-old farmer develops fever, chills, and a cough. He is treated by his local physician with
tetracycline for 1 week and feels improved. However, a week later he again develops a low-grade
fever associated with myalgias. Over the next few days he has several episodes of palpitation and
chest tightness that are worse with exertion. He is referred to you for further evaluation. On
examination, he has a temperature of 99.5°F, pulse 90/min, respirations 16/min, blood pressure
130/85. His lungs are clear. There is no jugular venous distention and the carotid upstrokes are
normal. The left ventricular (LV) impulse is not displaced; S1 and S2 are normal. There is a grade II/VI
midsystolic murmur that increases with handgrip. A midsystolic click is also present. The rest of the
examination is unremarkable. Chest x-ray and ECG findings are normal. Laboratory findings include
hemoglobin (Hgb) 14.2 mg/dl; white blood cell count (WBC) 15,000/mm 3 with 80 percent polys, 3
percent bands, 17 percent lymphs; Na 140 mmol/L, Cl 100 mmol/L, K 5.0 mmol/L, and HCO 3 25
mmol/L; urinalysis: clear, pH 6.6, 1+ protein, no WBC, 2 to 3 RBC/hpf. What would you order now?

A. Exercise testing

B. Exercise thallium testing

88
C. Ambulatory Holter monitor

D. Blood cultures

E. 1-week course of erythromycin

291. True statements regarding the macrophage in the pathogenesis of atherosclerosis include all of
the following EXCEPT:

A. Macrophages demonstrate both a contractile and a synthetic state

B. Macrophages are the principal cells in the initial lesion of atherosclerosis, the fatty streak

C. Macrophages are derived from circulating monocytes

D. Macrophages are capable of taking up oxidized LDL

E. Hypercholesterolemia increases the adhesion of circulating monocytes to the endothelium

292. True statements regarding the clinical history of patients with acute myocardial infarction (MI)
include all of the following EXCEPT:

A. A clear precipitating factor or prodromal symptoms can be identified in 90 percent of patients


with acute MI

B. Between 20 and 60 percent of nonfatal MIs are unrecognized by the patient and are identified
only by a subsequent routine ECG

C. One third of patients with a prodrome have had symptoms for 1 to 4 weeks before
hospitalization

D. Over one half of patients with a transmural MI have nausea and vomiting

E. The peak frequency of MI onset is between 6 AM and noon

293. Each of the following statements regarding the use of percutaneous transluminal coronary
angioplasty (PTCA) as a primary therapy in acute MI is true EXCEPT:

A. PTCA can be performed safely during acute MI

B. The primary success rate for PTCA during acute MI is approximately 90 percent

89
C. The PAMI trial showed that patients randomized to primary PTCA had a lower incidence of
death or reinfarction by hospital discharge and at 6-month follow-up

D. When performed in experienced centers, hospital length of stay and follow-up costs are
significantly less than for patients treated with thrombolysis

E. Primary PTCA is associated with a worse outcome compared with thrombolysis for acute MI
patients presenting with cardiogenic shock

294. True statements about effusive-constrictive pericarditis include all of the following EXCEPT:

A. In effusive-constrictive pericarditis, removal of pericardial fluid by aspiration does not lead to a


normal right atrial pressure

B. Idiopathic or presumed viral pericarditis is a common cause of effusive-constrictive pericar-


ditis

C. Physical findings in effusive-constrictive pericarditis resemble chronic constrictive pericarditis


more than cardiac tamponade

D. The diagnosis of effusive-constrictive pericarditis is made by careful hemodynamic monitoring


before and after pericardiocentesis

E. Treatment of effusive-constrictive pericarditis consists of total parietal and visceral


pericardiectomy

295. The following statements regarding myocardial stunning are true EXCEPT:

A. It is a state of depressed myocardial function due to chronic hypoperfusion

B. Stunning can be global or regional

C. Stunning can follow cardiopulmonary bypass

D. Oxygen free radicals and excess intracellular calcium likely contribute to stunning

E. Stunning affects both systolic and diastolic function

296. True statements regarding the effect of medications on the lipid profile include all of the following
EXCEPT:

A. Nonselective beta-blockers increase high-density lipoprotein (HDL) levels

B. Thiazide diuretics increase triglyceride levels

90
C. Estrogen replacement therapy raises HDL and triglyceride levels

D. Immunosuppressive agents increase triglyceride levels

E. Protease inhibitors raise total cholesterol and triglyceride levels

297. True statements about the ECG in congenital heart disease include all of the following EXCEPT:

A. T-wave inversions may be seen in normal neonatal ECGs

B. By the time the infant is 72 hours of age, the T waves should be inverted in V 1 to V3

C. The presence of right ventricular hypertrophy suggests single ventricle or inversion of the
ventricles

D. An ECG pattern of MI may indicate an anomalous origin of a coronary artery

E. T-wave inversion in the lateral precordial leads may be observed in subendocardial ischemia

298. The following statements regarding abrupt vessel closure after balloon coronary angioplasty are
correct EXCEPT:

A. Abrupt closure complicates 10 to 15 percent of cases

B. Deep dissection leading to thrombosis is the primary mechanism

C. The risk is higher in the setting of an acute coronary syndrome

D. It is associated with a mortality rate of approximately 4 percent and an MI rate of


approximately 20 percent

E. The use of stents and platelet glycoprotein IIb/IIIa inhibitors has reduced the incidence of
abrupt closure

299. True statements about the progression of atherosclerosis in venous aortocoronary artery bypass
grafts include all of the following EXCEPT:

A. Between 12 and 20 percent of vein grafts are occluded by the end of the first year following
surgery

B. At 10 years, the overall occlusion rate for a saphenous vein graft approaches 50 percent

91
C. The atherosclerotic process that occurs in venous grafts is histologically different from that
which occurs in native arterial vessels

D. Progression of disease in native coronary arteries occurs at a rate of 18 to 38 percent over the
first decade after operation

E. The annual rate of saphenous vein graft occlusion after the first year is only on the order of 2
percent

300. All of the following statements regarding ventricular septal defect (VSD) are true EXCEPT:

A. It is the most common form of congenital heart disease in infants and children

B. The most common VSD occurs in the membranous septum

C. Spontaneous VSD closure occurs in 45 percent of patients by the age of 3 years

D. Spontaneous closure occurs primarily by continued growth of the muscular septum

E. Complete heart block following surgical repair is uncommon

92

Potrebbero piacerti anche